+7 495 120-13-73 | 8 800 500-97-74

(для регионов бесплатно)

Содержание

Параллельная схема: характеристики, преимущества и недостатки

Параллельное соединение проводников

Параллельным соединением проводников называется такое соединение, когда начала всех проводников соединены в одну точку, а концы проводников – в другую точку (рисунок 4). Начало цепи присоединяется к одному полюсу источника напряжения, а конец цепи – к другому полюсу.

Рисунок 4. Схема параллельного соединения проводников

Из рисунка видно, что при параллельном соединении проводников для прохождения тока имеется несколько путей. Ток, протекая к точке разветвления А, растекается далее по трем сопротивлениям и равен сумме токов, уходящих от этой точки:

I = I1 + I2 + I3.

Если токи, приходящие к точке разветвления, считать положительными, а уходящие – отрицательными, то для точки разветвления можно написать:

то есть алгебраическая сумма токов для любой узловой точки цепи всегда равна нулю. Это соотношение, связывающее токи в любой точке разветвления цепи, называется первым законом Кирхгофа. Определение первого закона Кирхгофа может звучать и в другой формулировке, а именно: сумма токов втекающих в узел электрической цепи равна сумме токов вытекающих из этого узла.

Видео 2. Первый закон Кирхгофа

Обычно при расчете электрических цепей направление токов в ветвях, присоединенных к какой либо точке разветвления, неизвестны. Поэтому для возможности самой записи уравнения первого закона Кирхгофа нужно перед началом расчета цепи произвольно выбрать так называемые положительные направления токов во всех ее ветвях и обозначить их стрелками на схеме.

Пользуясь законом Ома, можно вывести формулу для подсчета общего сопротивления при параллельном соединении потребителей.

Общий ток, приходящий к точке А, равен:

Токи в каждой из ветвей имеют значения:

По формуле первого закона Кирхгофа

I = I1 + I2 + I3

или

Вынося U в правой части равенства за скобки, получим:

Сокращая обе части равенства на U, получим формулу подсчета общей проводимости:

или

g = g1 + g2 + g3.

Таким образом, при параллельном соединении увеличивается не сопротивление, а проводимость.

Пример 3. Определить общее сопротивление трех параллельно включенных сопротивлений, если r1 = 2 Ом, r2 = 3 Ом, r3 = 4 Ом.

откуда

Пример 4. Пять сопротивлений 20, 30 ,15, 40 и 60 Ом включены параллельно в сеть. Определить общее сопротивление:

откуда

Следует заметить, что при подсчете общего сопротивления разветвления оно получается всегда меньше, чем самое меньшее сопротивление, входящее в разветвление.

Если сопротивления, включенные параллельно, равны между собой, то общее сопротивление r цепи равно сопротивлению одной ветви r1, деленному на число ветвей n:

Пример 5. Определить общее сопротивление четырех параллельно включенных сопротивлений по 20 Ом каждое:

Для проверки попробуем найти сопротивление разветвления по формуле:

откуда

Как видим, ответ получается тот же.

Пример 6. Пусть требуется определить токи в каждой ветви при параллельном их соединении, изображенном на рисунке 5, а.

Рисунок 5. К примеру 6

Найдем общее сопротивление цепи:

откуда

Теперь все разветвления мы можем изобразить упрощенно как одно сопротивление (рисунок 5, б).

Падение напряжения на участке между точками А и Б будет:

U = I × r = 22 × 1,09 = 24 В.

Возвращаясь снова к рисунку 5, а видим, что все три сопротивления окажутся под напряжением 24 В, так как они включены между точками А и Б.

Рассматривая первую ветвь разветвления с сопротивлением r1, мы видим, что напряжение на этом участке 24 В, сопротивление участка 2 Ом. По закону Ома для участка цепи ток на этом участке будет:

Ток второй ветви

Ток третьей ветви

Проверим по первому закону Кирхгофа

I = I1 + I2 + I3 = 12 + 6 + 4 = 22 А.

Следовательно, задача решена верно.

Обратим внимание на то, как распределяются токи в ветвях нашего параллельного соединения. Первая ветвь: r1 = 2 Ом, I1 = 12 А

Вторая ветвь: r2 = 4 Ом, I2 = 6 А. Третья ветвь: r3 = 6 Ом, I3 = 4 А

Первая ветвь: r1 = 2 Ом, I1 = 12 А. Вторая ветвь: r2 = 4 Ом, I2 = 6 А. Третья ветвь: r3 = 6 Ом, I3 = 4 А.

Как видим, сопротивление первой ветви в два раза меньше сопротивление второй ветви, а ток первой ветви в два раза больше тока второй ветви. Сопротивление третьей ветви в три раза больше сопротивления первой ветви, а ток третьей ветви в три раза меньше тока первой ветви. Отсюда можно сделать вывод, что токи в ветвях при параллельном соединении распределяются обратно пропорционально сопротивлениям этих ветвей. Таким образом, по ветви с большим сопротивлением потечет ток меньший, чем по ветви с малым сопротивлением.

Для двух параллельных ветвей можно также, конечно, пользоваться данной выше формулой.

Однако общее сопротивление проводника при параллельном соединении в этом случае легче подсчитать по формуле:

или окончательно:

Последовательное соединение ламп накаливания.

Последовательное соединение ламп накаливания в домашнем быту используется редко. В свое время я подключал две лампы последовательно у себя в подъезде, но это был единичный случай.

Тут ситуация была такая, что подъездная лампа перегорала с периодичностью в один месяц, и надо было что-то делать.

Обычно, в таких случаях лампу включают через диод, чтобы она питалась пониженным напряжением 110В и долго служила. Вариант проверенный, но при этом сама лампа мерцает, да и светит в полнакала.

Когда же стоят две последовательно, то они так же питаются пониженным напряжением 110В, не мерцают, долго служат, светят и потребляют энергии как одна. Причем их можно развести по разным углам помещения, что тоже плюс.Но повторюсь – это редкий случай.

Посмотрите на рисунок ниже. Здесь изображены две схемы последовательного соединения ламп накаливания. В верхней части рисунка показана принципиальная схема, а в нижней части – монтажная. Причем для лучшего восприятия, монтажная схема показана с реальным изображением ламп и двужильного провода.

Здесь в линии коричневого цвета, лампы HL1 и HL2 соединены последовательно – одна за другой. Поэтому такое соединение называют последовательным.

Если подать напряжение питания 220В на концы L и N, то загорятся обе лампы, но гореть они будут не в полную силу, а в половину накала. Так как сопротивление нитей ламп рассчитано на питающее напряжение 220В, и когда они стоят в цепи последовательно, одна за другой, то за счет добавления сопротивления нити накала следующей лампы, общее сопротивление цепи будет увеличиваться, а значит, для следующей лампы напряжение всегда будет меньше согласно закону Ома.

Поэтому при последовательном соединении двух ламп напряжение 220В будет делиться пополам, и составит 110В для каждой.

На следующем рисунке показаны три лампы соединенные последовательно.

На этой схеме напряжение на каждой лампе составит около 73 Вольт, так как будет делиться уже между тремя лампами.

Так же примером последовательного соединения могут служить новогодние гирлянды. Здесь из миниатюрных лампочек с низким питанием создается одна лампа на напряжение 220В.

Например, берем лампочки, рассчитанные на 6,3 Вольта и делим их на 220 Вольт. Получается 35 штук. То есть, чтобы сделать одну лампу на напряжение 220В, нам нужно соединить последовательно 35 штук с напряжением питания 6,3 Вольта.

P.S. Так как напряжение в сети не постоянно, то расчет лучше производить исходя из 245 – 250 Вольт.

Как Вы знаете, у гирлянд есть один недостаток. Перегорает одна из ламп, например, канала зеленого цвета, значит, не горит канал зеленого цвета. Тогда мы идем на базар, покупаем лампочки зеленого цвета, а потом дома по одной вынимаем, вставляем новую, и пока не заработает канал, перебираем его весь.

Вывод:

Недостатком последовательного соединения является то, что если выйдет из строя хоть одна из ламп, гореть не будут все, так как нарушается электрическая цепь.

А вторым недостатком, как Вы уже догадались, является слабое свечение. Поэтому последовательное соединение ламп накаливания на напряжение 220В в домашних условиях практически не применяется.

Первый закон Кирхгофа

Как я уже упоминал, законы Кирхгофа вместе с законом Ома являются основными при анализе и расчётах электрических цепей. Закон Ома был подробно рассмотрен в двух предыдущих статьях, теперь настала очередь для законов Кирхгофа. Их всего два, первый описывает соотношения токов в электрических цепях, а второй – соотношение ЭДС и напряжениями в контуре. Начнём с первого.

Первый закон Кирхгофа гласит, что алгебраическая сумма токов в узле равна нулю. Описывается это следующим выражением

где ∑ — обозначает алгебраическую сумму.

Слово «алгебраическая» означает, что токи необходимо брать с учётом знака, то есть направления втекания. Таким образом, всем токам, которые втекают в узел, присваивается положительный знак, а которые вытекают из узла – соответственно отрицательный. Рисунок ниже иллюстрирует первый закон Кирхгофа

На рисунке изображен узел, в который со стороны сопротивления R1 втекает ток, а со стороны сопротивлений R2, R3, R4 соответственно вытекает ток, тогда уравнение токов для данного участка цепи будет иметь вид

Первый закон Кирхгофа применяется не только к узлам, но и к любому контуру или части электрической цепи. Например, когда я говорил о параллельном соединении приемников энергии, где сумма токов через R1, R2 и R3 равна втекающему току I.

Примеры использования

  • Батареи гальванических элементов или аккумуляторов, в которых отдельные химические источники тока соединены последовательно (для увеличения напряжения) или параллельно (для увеличения тока).
  • Регулировка мощности электрического устройства, состоящего из нескольких одинаковых потребителей электроэнергии, путём их переключения с параллельного на последовательное соединение. Таким способом регулируется мощность конфорки электрической плиты, состоящей из нескольких спиралей; мощность (скорость движения) электровоза, имеющего несколько тяговых двигателей.
  • Делитель напряжения
  • Балласт
  • Шунт

Какой способ лучше?

Метод «шлейфов» не слишком удобен только тем, что любой потребитель по цепи зависит от предыдущего. Например, если произойдёт обрыв провода на второй розетке, то третья и четвёртая также останутся без напряжения. Но при этом нельзя не выделить экономию проводника при начальном монтаже электропроводки.

Рисунок 3: Комбинированное соединение розеток

К тому же, «шлейфом» очень удобно проводить линии, когда необходимо минимизировать количество штроб в стенах. А делают это при монтаже проводки по полу или потолку, в специальной гофрированной трубе. Тогда остаётся провести только основные штробы к розеткам и между ними.

Вывод: прокладка электропроводки «шлейфом» удобна и экономична, не занимает много времени в процессе монтажа, имеет длительный эксплуатационный срок и совсем незначительные недостатки, которые можно оставить без внимания.

Последовательное соединение ламп накаливания.

Последовательное соединение ламп накаливания в домашнем быту используется редко. В свое время я подключал две лампы последовательно у себя в подъезде, но это был единичный случай.

Тут ситуация была такая, что подъездная лампа перегорала с периодичностью в один месяц, и надо было что-то делать.

Обычно, в таких случаях лампу включают через диод, чтобы она питалась пониженным напряжением 110В и долго служила. Вариант проверенный, но при этом сама лампа мерцает, да и светит в полнакала.

Когда же стоят две последовательно, то они так же питаются пониженным напряжением 110В, не мерцают, долго служат, светят и потребляют энергии как одна. Причем их можно развести по разным углам помещения, что тоже плюс.Но повторюсь – это редкий случай.

Посмотрите на рисунок ниже. Здесь изображены две схемы последовательного соединения ламп накаливания. В верхней части рисунка показана принципиальная схема, а в нижней части – монтажная. Причем для лучшего восприятия, монтажная схема показана с реальным изображением ламп и двужильного провода.

Здесь в линии коричневого цвета, лампы HL1 и HL2 соединены последовательно – одна за другой. Поэтому такое соединение называют последовательным.

Если подать напряжение питания 220В на концы L и N, то загорятся обе лампы, но гореть они будут не в полную силу, а в половину накала. Так как сопротивление нитей ламп рассчитано на питающее напряжение 220В, и когда они стоят в цепи последовательно, одна за другой, то за счет добавления сопротивления нити накала следующей лампы, общее сопротивление цепи будет увеличиваться, а значит, для следующей лампы напряжение всегда будет меньше согласно закону Ома.

Поэтому при последовательном соединении двух ламп напряжение 220В будет делиться пополам, и составит 110В для каждой.

На следующем рисунке показаны три лампы соединенные последовательно.

На этой схеме напряжение на каждой лампе составит около 73 Вольт, так как будет делиться уже между тремя лампами.

Так же примером последовательного соединения могут служить новогодние гирлянды. Здесь из миниатюрных лампочек с низким питанием создается одна лампа на напряжение 220В.

Например, берем лампочки, рассчитанные на 6,3 Вольта и делим их на 220 Вольт. Получается 35 штук. То есть, чтобы сделать одну лампу на напряжение 220В, нам нужно соединить последовательно 35 штук с напряжением питания 6,3 Вольта.

P.S. Так как напряжение в сети не постоянно, то расчет лучше производить исходя из 245 – 250 Вольт.

Как Вы знаете, у гирлянд есть один недостаток. Перегорает одна из ламп, например, канала зеленого цвета, значит, не горит канал зеленого цвета. Тогда мы идем на базар, покупаем лампочки зеленого цвета, а потом дома по одной вынимаем, вставляем новую, и пока не заработает канал, перебираем его весь.

Вывод:

Недостатком последовательного соединения является то, что если выйдет из строя хоть одна из ламп, гореть не будут все, так как нарушается электрическая цепь.

А вторым недостатком, как Вы уже догадались, является слабое свечение. Поэтому последовательное соединение ламп накаливания на напряжение 220В в домашних условиях практически не применяется.

Как выглядит формула Георга Ома

Примером такого типа подключения резисторов может быть соединение цепи потребителей электроэнергии в многоквартирном доме. Так, светодиоды, отопительный радиатор, микроволновка и другие приборы установлены в цепи параллельно.

Вольтметр, который подключают в цепь, будет показывать напряжение на всех резисторах. Тогда оно везде будет равным и формулу можно записать как:

U1 = U2 = U.

Схема параллельного соединения

Когда образуются ветви при подключении, то часть общего напряжения проходит через первый резистор, а часть — через второй и так далее. Поэтому при таком виде соединения резисторов Fтока в неразветвлённой точке будет равняться суммарной Fтока в отдельных резисторах и записывается как:

I = I1 + I2.

Расчет силы тока при помощи закона Ома записывается как:

I = U/R;

I1 = U1/R1;

I2 = U2/R2.

Из формулы следует:

U/R = U1/R1 + U2/R2;

U = U1 = U2;

1/R = 1/R1 + 1/R2.

Дословно правило звучит так: число, обратное общему сопротивлению при параллельном подключении, будет суммарно равно числу обратного сопротивления.

Зависимость сопротивления

Значение электропроводимости зависит от нескольких факторов, которые необходимо учитывать при расчетах, изготовлении элементов резистивной нагрузки (резисторов), ремонте и проектировании устройств. К этим факторам необходимо отнести следующие:

  1. Температура окружающей среды и материала.
  2. Электрические величины.
  3. Геометрические свойства вещества.
  4. Тип материала, из которого изготовлен проводник (полупроводник).

Электрические величины

Зависимость величины электропроводимости от параметров электричества определяется законом Ома. Существует две формулировки: одна — для участка, а другая — для полной цепи. В первом случае соотношение определяются, исходя из значений силы тока (I) и напряжения (U) простой формулой: I = U / R. Из соотношения видна прямо пропорциональная зависимость тока от величины напряжения, а также обратно пропорциональная от сопротивления. Можно выразить R: R = U / I.

Для расчета электропроводимости всего участка следует воспользоваться соотношением между ЭДС (e), силой тока (i), а также внутренним сопротивлением источника питания (Rвн): i = e / (R+Rвн). В этом случае величина R вычисляется по формуле: R = (e / i) — Rвн. Однако при выполнении расчетов необходимо учитывать также геометрические параметры и тип проводника, поскольку они могут существенно повлиять на вычисления.

Тип и геометрические параметры

Свойство вещества к проводимости электричества определяется структурой кристаллической решетки, а также количеством свободных носителей. Исходя из этого, тип вещества является ключевым фактором, который определяет величину электропроводимости. В науке коэффициент, определяющий тип вещества, обозначается литерой «р» и называется удельным сопротивлением. Его значение для различных материалов (при температуре +20 градусов по Цельсию) можно найти в специальных таблицах.

Иногда для удобства расчетов используется обратная величина, которая называется удельной проводимостью (σ). Она связана с удельным сопротивлением следующим соотношением: p = 1 / σ. Площадь поперечного сечения (S) влияет на электрическое сопротивление. С физической точки зрения, зависимость можно понять следующим образом: при малом сечении происходят более частые взаимодействия частиц электрического тока с узлами кристаллической решетки. Поперечное сечение можно вычислить по специальному алгоритму:

  1. Измерение геометрических параметров проводника (диаметр или длину сторон) при помощи штангенциркуля.
  2. Визуально определить форму материала.
  3. Вычислить площадь поперечного сечения по формуле, найденной в справочнике или интернете.

В случае когда проводник имеет сложную структуру, необходимо вычислить величину S одного элемента, а затем умножить результат на количество элементов, входящих в его состав. Например, если провод является многожильным, то следует вычислить S для одной жилы. После этого нужно умножить, полученную величину S, на количество жил. Зависимость R от вышеперечисленных величин можно записать в виде соотношения: R = p * L / S. Литера «L» является длиной проводника. Однако для получения точных расчетов необходимо учитывать температурные показатели внешней среды и проводника.

Температурные показатели

Существует доказательство зависимости удельного сопротивления материала от температуры, основанное на физическом эксперименте. Для проведения опыта нужно собрать электрическую цепь, состоящую из следующих элементов: источника питания, нихромовой спирали, соединительных проводов амперметра и вольтметра. Приборы нужны для измерения значений силы тока и напряжения соответственно. При протекании электричества происходит нагревание нихромовой пружины. По мере ее нагревания, показания амперметра уменьшаются. При этом происходит существенное падение напряжения на участке цепи, о котором свидетельствуют показания вольтметра.

В радиотехнике уменьшение величины напряжение называется просадкой или падением. Формула зависимости р от температуры имеет следующий вид: p = p0 * . Значение p0 — удельное сопротивление материала, взятого из таблицы, а литера «t» — температура проводника.

Температурный коэффициент «а» принимает следующие значения: для металлов — a>0, а для электролитических растворов — a<0. Для получения формулы, определяющей все зависимости, необходимо подставить все соотношения в общую формулу зависимости R от типа материала, температуры, длины и сечения: R = p0 * * L / S. Формулы используются только для расчетов и изготовления резисторов. Для быстрого измерения величины сопротивления применяется омметр.

Формула параллельного соединения резисторов

Общее сопротивление нескольких резисторов соединенных параллельно определяется по следующей формуле:

Ток, протекающий через отдельно взятый резистор, согласно закону Ома, можно найти по формуле:

Пример  №1

При разработке устройства, возникла необходимость установить резистор с сопротивлением 8 Ом. Если мы просмотрим весь номинальный ряд стандартных значений резисторов, то мы увидим, что резистора с сопротивлением в 8 Ом в нем нет.

Выходом из данной ситуации будет использование двух параллельно соединенных резисторов. Эквивалентное значение сопротивления для двух резисторов соединенных параллельно рассчитывается следующим образом:

Данное уравнение показывает, что если R1 равен R2, то сопротивление R составляет половину сопротивления одного из двух резисторов. При R = 8 Ом, R1 и R2 должны, следовательно, иметь значение 2 × 8 = 16 Ом.
Теперь проведем проверку, рассчитав общее сопротивление двух резисторов:

Таким образом, мы получили необходимое сопротивление 8 Ом, соединив параллельно два резистора по 16 Ом.

Пример расчета №2

Найти общее сопротивление  R из трех параллельно соединенных резисторов:

Общее сопротивление R рассчитывается по формуле:

Этот метод расчета может быть использованы для расчета любого количества отдельных сопротивлений соединенных параллельно.

Один важный момент, который необходимо запомнить при расчете параллельно соединенных резисторов – это то, что общее сопротивление всегда будет меньше, чем значение наименьшего сопротивления в этой комбинации.

Как рассчитать сложные схемы соединения резисторов

Более сложные соединения резисторов могут быть рассчитаны путем систематической группировки резисторов. На рисунке ниже необходимо посчитать общее сопротивление цепи, состоящей из трех резисторов:

Резисторы R2 и R3 соединены последовательно (группа 2). Они в свою очередь соединены параллельно с резистором R1 (группа 1).

Последовательное соединение резисторов группы 2 вычисляется как сумма сопротивлений R2 и R3:

В результате мы упрощаем схему в виде двух параллельных резисторов. Теперь общее сопротивление всей схемы можно посчитать следующим образом:

Расчет более сложных соединений резисторов можно выполнить используя законы Кирхгофа.

Ток, протекающий в цепи параллельно соединенных резисторах

Общий ток I протекающий в цепи параллельных резисторов равняется сумме отдельных токов, протекающих во всех параллельных ветвях, причем ток в отдельно взятой ветви не обязательно должен быть равен току в соседних ветвях.

Несмотря на параллельное соединение, к каждому резистору приложено одно и то же напряжение. А поскольку величина сопротивлений в параллельной цепи может быть разной, то и величина протекающего тока через каждый резистор тоже будет отличаться (по определению закона Ома).

Рассмотрим это на примере двух параллельно соединенных резисторов. Ток, который течет через каждый из резисторов ( I1 и I2 ) будет отличаться друг от друга поскольку сопротивления резисторов R1 и R2 не равны.
Однако мы знаем, что ток, который поступает в цепь в точке «А» должен выйти из цепи в точке «B» .

Первое правило Кирхгофа гласит: «Общий ток, выходящий из цепи равен току входящий в цепь».

  • Таким образом, протекающий общий ток в цепи  можно определить как:
  • I = I1 + I2
  • Затем с помощью закона Ома можно вычислить ток, который протекает через каждый резистор:
  • Ток, протекающий в R1 = U ÷ R1 = 12 ÷ 22 кОм = 0,545 мА
  • Ток, протекающий в R 2 = U ÷ R2 = 12 ÷ 47 кОм = 0,255 мА
  • Таким образом, общий ток будет равен:
  • I = 0,545 мА + 0,255 мА = 0,8 мА
  • Это также можно проверить, используя закон Ома:
  • I = U ÷ R = 12 В ÷ 15 кОм = 0,8 мА (то же самое)
  • где 15кОм — это общее сопротивление двух параллельно соединенных резисторов (22 кОм и 47 кОм)
  • И в завершении хочется отметить, что большинство современных резисторов маркируются цветными полосками и назначение ее можно узнать здесь.

Параллельное соединение резисторов — онлайн калькулятор

Чтобы быстро вычислить общее сопротивление двух и более резисторов, соединенных параллельно, вы можете воспользоваться следующим онлайн калькулятором:

Подведем итог

Когда два или более резистора соединены так, что оба вывода одного резистора соединены с соответствующими выводами другого резистора или резисторов, то говорят, что они соединены между собой параллельно. Напряжение на каждом резисторе внутри параллельной комбинации одинаковое, но токи, протекающие через них, могут отличаться друг от друга, в зависимости от величины сопротивлений каждого резистора.

Эквивалентное или полное сопротивление параллельной комбинации всегда будет меньше минимального сопротивления резистора входящего в параллельное соединение.

Последовательное соединение источников питания

Теперь давайте представим вот такую ситуацию. Что будет, если в нашей обрезанной водобашне полной воды добавим еще одну такую же сверху полную воды? Схематически это будет выглядеть примерно вот так:

Как вы думаете, уменьшится давление на землю, или увеличится? Понятное дело, что увеличится! Да еще и ровно в два раза! Почему так произошло? Уровень воды стал выше, следовательно, давление на дно увеличилось.

Если “минус” одной батарейки соединить с “плюсом” другой батарейки, то их общее напряжение суммируется.

Полностью заряженная батарейка будет выглядеть как башня, полностью залитая водой с учетом того, что работает насос автоматической подачи воды. По аналогии, насос – это ЭДС.

Наполовину разряженная батарейка будет уже выглядеть примерно вот так:

Можно сказать, насос уже не справляется.

Батарейка посаженная в “ноль” будет выглядеть вот так:

Насос автоматической подачи воды сломался.

Естественно, что если вы соедините полностью заряженную и наполовину дохлую батарейку последовательно, то их общее напряжение будет выглядеть примерно вот так:

Давайте все это продемонстрируем на практике. Итак, у нас есть 2 литий-ионных аккумулятора. Я их пометил цифрами 1 и 2. С плюса каждого аккумулятора я вывел красный провод, а с минуса – черный.

Давайте замеряем напряжение аккумулятора под №1 с помощью мультиметра. Как это сделать, я еще писал в статье Как измерить ток и напряжение мультиметром.

На первом аккумуляторе у нас напряжение 3,66 Вольт. Это типичное значение литий-ионного аккумулятора.

Таким же способом замеряем напряжение на аккумуляторе №2

О, как совпало). Те же самые 3,66 Вольт.

Для того, чтобы соединить последовательно эти аккумуляторы, нам надо сделать что-то подобное:

Также как и в башнях, нам надо соединить основание одной башни с верхушкой другой башни. В источниках питания, типа аккумуляторов или батареек, нам надо соединить минус одной батарейки с плюсом другой. Так мы и сделаем. Соединяем плюс одной батарейки с минусом другой и получаем… сумму напряжений каждой батарейки! Как вы помните, на первой батарейке у нас было напряжение 3,66 В, на второй тоже 3,66 В. 3,66+3,6=7,32 В.

Мультиметр показывает 7,33 В. 0,01В спишем на погрешность измерений.

Это свойство прокатывает не только с двумя аккумуляторами, но также с их бесконечным множеством. Думаю, не стоит говорить, что если выставить в ряд штук 100 таких аккумуляторов, соединить последовательно и коснуться крайних полюсов голыми руками, то все это может завершиться даже летальным исходом.

Оцените статью:

Элеком37, Закон Ома. Последовательное и параллельное соединение проводников.

Закон Ома. Последовательное и параллельное соединение проводников.

Немецкий физик Г.Ом в 1826 году экспериментально установил, что сила тока I, текущего по однородному металлическому проводнику (то есть проводнику, в котором не действуют сторонние силы) сопротивлением R, пропорциональна напряжению U на концах проводника:

Величину R принято называть электрическим сопротивлением. Проводник, обладающий электрическим сопротивлением, называется резистором. Это соотношение выражает закон Ома для однородного участка цепи: сила тока в проводнике прямо пропорциональна приложенному напряжению и обратно пропорциональна сопротивлению проводника.

Проводники, подчиняющиеся закону Ома, называются линейными. Графическая зависимость силы тока I от напряжения U (такие графики называются вольт-амперными характеристиками, сокращенно ВАХ) изображается прямой линией, проходящей через начало координат.

Следует отметить, что существует много материалов и устройств, не подчиняющихся закону Ома, например, полупроводниковый диод или газоразрядная лампа. Даже у металлических проводников при достаточно больших токах наблюдается отклонение от линейного закона Ома, так как электрическое сопротивление металлических проводников растет с ростом температуры.

Проводники в электрических цепях можно соединять двумя способами: последовательно и параллельно. У каждого способа есть свои закономерности.

1. Закономерности последовательного соединения:

Формула для общего сопротивления последовательно соединенных резисторов справедлива для любого числа проводников. Если же в цепь последовательно включено n одинаковых сопротивлений R, то общее сопротивление R0 находится по формуле:

2. Закономерности параллельного соединения:

Формула для общего сопротивления параллельно соединенных резисторов справедлива для любого числа проводников. Если же в цепь параллельно включено n одинаковых сопротивлений R, то общее сопротивление R0 находится по формуле:

Электроизмерительные приборы

Для измерения напряжений и токов в электрических цепях постоянного тока используются специальные приборы – вольтметры и амперметры.

Вольтметр предназначен для измерения разности потенциалов, приложенной к его клеммам. Он подключается параллельно участку цепи, на котором производится измерение разности потенциалов. Любой вольтметр обладает некоторым внутренним сопротивлением RB. Для того чтобы вольтметр не вносил заметного перераспределения токов при подключении к измеряемой цепи, его внутреннее сопротивление должно быть велико по сравнению с сопротивлением того участка цепи, к которому он подключен.

Амперметр

предназначен для измерения силы тока в цепи. Амперметр включается последовательно в разрыв электрической цепи, чтобы через него проходил весь измеряемый ток. Амперметр также обладает некоторым внутренним сопротивлением RA. В отличие от вольтметра, внутреннее сопротивление амперметра должно быть достаточно малым по сравнению с полным сопротивлением всей цепи.

Урок 29. закон ома для участка цепи. соединения проводников — Физика — 10 класс

Физика, 10 класс

Урок 29. Закон Ома для участка цепи. Соединения проводников

Перечень вопросов, рассматриваемых на уроке:

  1. условия, необходимые для существования электрического тока;
  2. постоянный электрический ток;
  3. закон Ома для участка цепи;
  4. формула расчета сопротивления проводника с учетом свойств материала проводника и его геометрических размеров;
  5. типы соединений проводников и формулы расчета параметров электрической цепи для каждого типа.

Глоссарий по теме.

Сила тока I — скалярная величина, равная отношению заряда q, прошедшего через поперечное сечение проводника, к промежутку времени t, в течение которого шёл ток.

Постоянный ток — электрический ток, не изменяющийся со временем.

Последовательное соединение проводников.

При последовательном соединении электрическая цепь не имеет разветвлений. Все проводники включают в цепь поочередно друг за другом.

Параллельное соединение проводников. При параллельном соединении концы проводников присоединены к одной и той же паре точек.

Смешанное соединение проводниковэто такое соединение, когда в цепи присутствует и последовательное, и параллельное соединение.

Узел – это точка электрической цепи, где сходится не менее трех ветвей.

Свойство проводника ограничивать силу тока в цепи, то есть противодействовать электрическому току, называют электрическим сопротивлением проводника.

Резистор или проводник

элемент электрических цепей, обладающий определённым или переменным значением электрического сопротивления.

Основная и дополнительная литература по теме урока:

1. Мякишев Г.Я., Буховцев Б.Б., Сотский Н.Н. Физика.10 класс. Учебник для общеобразовательных организаций М.: Просвещение, 2017. – С. 335 – 340.

2. Рымкевич А.П. Сборник задач по физике. 10-11 класс. — М.: Дрофа, 2009. – С. 105 – 109.

3. Элементарный учебник физики. Учебное пособие в 3 томах под редакцией академика Ландсберга Г.С.: Т.2. Электричество и магнетизм. – 12-е изд. – М.: ФИЗМАТЛИТ, 2001. С. 110 – 115.

4. Тульчинский М.Е. Качественные задачи по физике в средней школе. Пособие для учителей. Изд. 4-е, переработ. и доп. М. «Просвещение», 1972. С. 83 – 87.

5. Савельев И.В. Курс общей физики, том II. Электричество. М.: Изд. «Наука», 1970 г. С. 108.

Открытые электронные ресурсы:

http://kvant.mccme.ru/1979/02/elektrichestvo_ie_temperatura.htm

Теоретический материал для дополнительного изучения

Сложно представить нашу жизнь без электрического тока. Каждый день, не задумываясь, мы используем различные электрические приборы, в основе работы которых лежат простые и сложные электрические цепи.

Какому закону подчиняются основные параметры электрических цепей? Как рассчитать эти цепи, чтобы приборы работали исправно?

Вы уже знаете, электрическим током называют упорядоченное (направленное) движение заряженных частиц.

Для возникновения и существования электрического тока в проводнике необходимо:

  1. наличие свободных заряженных частиц;
  2. сила, действующая на них в определённом направлении, то есть наличие электрического поля в проводнике.

Различают следующие действия электрического тока:

  1. тепловое ;
  2. химическое ;
  3. магнитное .

Постоянный ток — электрический ток, у которого сила тока и направление не изменяются со временем.

Сила тока I равна отношению электрического заряда q, прошедшего через поперечное сечение проводника, ко времени его прохождения t:

За направление электрического тока условно выбрано направление движения положительно заряженных частиц, то есть в сторону, противоположную направлению движения электронов.

Для каждого проводника – твердого, жидкого и газообразного – существует определённая зависимость силы тока от приложенной разности потенциалов (напряжения) на концах проводника. Эту зависимость выражает, так называемая,

вольт-амперная характеристика проводника.

Для широкого класса проводников (в т. ч. металлов ) при неизменной температуре справедлив закон Ома для участка цепи:

Сила тока на участке цепи прямо пропорциональна приложенному напряжению U и обратно пропорциональна сопротивлению этого участка цепи:

Закон имеет простую форму, но доказать экспериментально его справедливость довольно трудно.

Закон Ома является основой всей электротехники постоянных токов. Из закона Ома вытекает, что замыкать обычную осветительную сеть проводником малого сопротивления опасно.

Основная электрическая характеристика проводника – сопротивление. От этой величины зависит сила тока в проводнике при заданном напряжении. Причиной электрического сопротивления является взаимодействие электронов при их движении по проводнику с ионами кристаллической решетки.

Сопротивление проводника зависит от свойств материала проводника и его геометрических размеров.

Электрическое сопротивление металлов прямо пропорционально длине проводника и обратно пропорционально площади его поперечного сечения:

где величина ρ – удельное сопротивление проводника — величина, зависящая от рода вещества и его состояния (от температуры в первую очередь). Удельное сопротивление веществ приводятся в справочных таблицах.

Омметр – прибор для измерения сопротивления.

От источника тока энергия может быть передана по проводам к устройствам, потребляющим энергию. Для этого составляют электрические цепи различной сложности. Различают последовательное, параллельное, смешанное соединения проводников.

Последовательное соединение проводников. При последовательном соединении электрическая цепь не имеет разветвлений. Все проводники включают в цепь поочередно друг за другом. Главная особенность последовательного соединения заключается в том, что через все проводники протекает одинаковый ток. Если через один проводник протекает ток определенной величины, то такой же ток протекает и через все остальные. Если хотя бы в одном проводнике отсутствует ток, то он обязательно отсутствует и во всех остальных. Напряжение на концах последовательно соединенных проводников складывается. Полное сопротивление всего участка цепи при последовательном соединении равно сумме сопротивлений всех проводников.

Последовательное соединение

Физическая величина

Формула

Сила тока

I = I1 = I2

Напряжение

U = U1 + U2

Сопротивление

R = R1 + R2

Параллельное соединение проводников. При параллельном соединении концы проводников присоединены к одной и той же паре точек.

Параллельное соединение

Физическая величина

Формула

Сила тока

I = I1 + I2

Напряжение

U = U1 = U2

Сопротивление

Узел – это точка электрической цепи, где сходится не менее трех ветвей.

Узел обозначается на схеме жирной точкой в том месте, где ветви соединяются между собой.

Смешанное соединение проводников.

Смешанным соединением проводников называют такое соединение, при котором в цепи присутствует и последовательное, и параллельное соединение.

Метод эквивалентных преобразований заключается в том, что электрическую цепь или ее часть заменяют более простой по структуре электрической цепью. При этом токи и напряжения в непреобразованной части цепи должны оставаться неизменными, т.е. такими, какими они были до преобразования. В результате преобразований расчет цепи упрощается и часто сводится к элементарным арифметическим операциям.

Расчет сопротивления сложной цепи:

Рези́стор или проводник — пассивный элемент электрических цепей, обладающий определённым или переменным значением электрического сопротивления.

Примеры и разбор решения заданий

1. Выберите один из 3 вариантов ответа:

При параллельном соединении проводников…

1) напряжение зависит от сопротивления на данном участке цепи

2) напряжение везде разное

3) напряжение везде одинаковое

Ответ: 3) напряжение везде одинаковое.

2. На участке цепи, изображенном на рисунке, сопротивление каждого из резисторов равно 24 Ом. Чему равно полное сопротивление участка при замкнутом ключе К?

Решение.

После замыкания ключа схема будет представлять собой параллельное соединение резистора с двумя последовательно соединенными резисторами.

Полное сопротивление участка при замкнутом ключе равно

(R+R)R/((R+R) + R) = 2R/3 = 16 Ом.

Ответ: 16 Ом.

Последовательное и параллельное соединение электрических сопротивлений

Рассмотрим пример электрической цепи, несколько более сложной, чем рассмотрен­ная ранее цепь с выключателем и лампочкой. Итак, обратимся к схеме на рис. 1. Имеется электрическая цепь, составленная из трех резисторов. Мы хотим узнать об­щее сопротивление между клеммами А и В.

Рис. 1   цепь с последовательно параллельным соединением резисторов

Итак, имеются три резистора: R1 соединен (включен) параллельно R2, а эта пара соединена последовательно с R3

Разобьем цепь на простейшие части, что всегда удобно при анализе сложных це­пей и схем. Если мы знаем метод определения величины резисторов, соединенных последовательно, можно использовать его для вычисления сопротивления, образуе­мого R3, соединенного последовательно с парой R1 и R2. Но так как мы еще не знаем способа нахождения величины сопротивления параллельного соединения, то сначала требуется разобраться с этим. Вопрос о последовательности анализа цепей и схем очень важен, и мы еще не раз будем к нему обращаться.

Если два резистора (или любых другие компонентов) соединены параллельно, то они должны иметь одинаковое падение напряжения на них. Следовательно, в качест­ве отправной точки, можно воспользоваться законом Ома.

I=V/R.

Теперь, применяя закон Кирхгофа о токах, можно смело утверждать что:

Iобщий= IR1+IR2+…

Значит:       V/Rпаралельное = V/R1 + V/R2 + …

Разделив на V, получим:      1/Rпаралельное = 1/R1 + 1/R2 + …

Итак: обратная величина общего параллельного сопротивления равна сумме об­ратных величин всех резисторов.

Величину, обратную сопротивлению, в электротехнике часто называют про­водимостью, и обозначают буквой С. Тогда

Gобщ =G1+G2

Таким образом, при параллельном включении элементов, алгебраически склады­ваются их проводимости.

В частном случае, когда параллельно включены два резистора, можно вывести более удобную формулу для расчета общего сопротивления (это выражение часто называют «произведение над суммой»):

Rпараллельное = R1R2/R1+R2

Итак, мы решили проблему вычисления параллельного включения сопротивле­ний, а теперь необходимо разобраться с последовательным включением.

Для начала, упростим схему, поскольку мы уже умеем вычислять общее сопротив­ление параллельного соединения, которое теперь можно заменить одним сопротив­лением соответствующей величины. Таким образом, на рис. 1.5 показан упрощенный вид все той же цепи, что и на рис. 2, но параллельное включение двух сопротивле­ний заменено одним, так называемым эквивалентным.

Используя закон Кирхгофа о напряжениях, легко сделать вывод, что сумма паде­ний напряжений на резисторах должна быть равна приложенной ЭДС:

Vобщее = VR1 + V R2 +…

Рис. 2 Упрощение схемы на рис. 1,используя эквивалентное сопротивление

Теперь применив закон Ома, получим:

VобщееI = IR1 + IR2 +…

Но поскольку мы пытаемся рассчитать эквивалентное сопротивление, величина которого равна общему сопротивлению, то удобнее записать

IRобщее = IR1 + I R2 +…

Откуда:

R последовательное=R1 + R2 +…

Общее сопротивление последовательно соединенных резисторов равно сумме со­противлений резисторов.

Используя формулы для параллельного и последовательного включения со­противлений, можно вычислить общее сопротивление любой сложной цепи, как бы «устрашающе» на первый взгляд она не выглядела.

Схемы могут быть очень сложными, но задача их анализа вполне разрешима, если подойти к ней логически. Здесь главное найти изначальный подход к решению, а ма­нипуляции с числами — проблема второстепенная.

Обратимся к цепи, изображенной на рис. 3. Требуется вычислить общее сопро­тивление цепи, то есть сопротивление между клеммами А и В. Поскольку цепь слож­ная, у нас нет правила для нахождения ее эквивалентного сопротивления напрямую, а значит, мы должны разбить сложную цепь на простейшую, к которой где можно применить уже известные нам правила. Таким образом, требуется выделить из слож­ной цепи группы компонентов, имеющие только последовательные или только парал­лельные соединения.

Рис. 3 Сложная разветвлённая цепь

В этом примере — между узлами А и D только параллельно включенные компо­ненты. Можно вычислить значение их эквивалентного сопротивления и подставить его в схему:

Rпараллельное = произведение / сумма = 6×12/6+12 = 4 Ом.

Теперь перечерчиваем схему, заменяя параллельное включение этихдвух элемен­тов их эквивалентным сопротивлением (рис. 4).

Рис. 4 Первое упрощение сложной цепи с рис. 3

Теперь имеются только последовательные и параллельные соединения между узлами А и С. Имеется выбор расчета — либо сперва рассчитать последовательное соединение 2 Ом и 4 Ом, либо параллельное соединение 3 Ом и 6 Ом. Рассчитаем сперва последовательное соединение, поскольку в результате получим эквивалент­ное сопротивление, включенное параллельно сопротивлениям 3 Ом и 6 Ом, а затем найдем сопротивление трех параллельно включенных резисторов.

Rпоследовательное = R1 + R2=4 + 2 = 6 Ом.

Снова перечерчиваем схему, заменив только что рассчитанное последовательное соединение двух резисторов, одним эквивалентным (рис. 1.8).

Рис. 5 Второе упрощение сложной цепи с рис. 3

Теперь имеются три компонента, включенные параллельно. Тогда:

1/R = 1/R1 + 1/R2 + 1/R3 = 1/3 + 1/6 + 1/6 = 2/3,

Rпаралельное=1,5 Ом

Теперь имеется совсем простая цепь, состоящая из двух последовательно вклю­ченных сопротивлений по 1,5 Ом. Применив правило для последовательного включе­ния двух резисторов, не трудно сказать, общее сопротивление всей цепи равно 3 Ом.

Итак, методами последовательного и параллельного соединения элементов, а так­же путем непосредственного применения законов Ома и Кирхгофа, можно анализиро­вать цепи любой сложности. Однако, существует ряд полезных методов, которые дают возможность несколько упростить и ускорить анализ электрических цепей и схем. Су­ществует множество различных учебников и книг по теории электрических цепей и электротехники, где разбираются различные методы анализа сложных цепей. Рассмат­ривать их все здесь вряд ли является целесообразным, поскольку цели настоящей книги несколько иные. Однако, к ряду таких методов мы будем обращаться далее, где и остано­вимся на них подробнее. Здесь же приведем некоторые самые общие рекомендации.

  • выбор отправной точки (то есть той группы элементов цепи, с которой начина­ется анализ сцепи (см., например, рассуждения к рис. 2) для решения задачи анализа цепи является очень важным и зачастую критическим;
  • отправную точку нужно стараться выбирать как можно дальше от внешних клемм цепи;
  • в качестве отправной точки выбирают группы элементов, включенные только последовательно или только параллельно;
  • анализ обычно проводится от в направлении от отправной точки к внешним клеммам;
  • Процессу анализа цепи очень помогает перечерчивание схемы, постепенно упрощая ее путем замены уже рассчитанной группы элементов на один эквива­

Морган Джонс. Ламповые усилителию. Перевод с английского под общей научной редакцией к.т.н. доц. Иванюшкина Р Ю.

Последовательное и параллельное соединение проводников

 

Если нам надо, чтобы электроприбор работал, мы должны подключить его к источнику тока. При этом ток должен проходить через прибор и возвращаться вновь к источнику, то есть цепь должна быть замкнутой.

Но подключение каждого прибора к отдельному источнику осуществимо, в основном, в лабораторных условиях. В жизни же приходится иметь дело с ограниченным количеством источников и довольно большим количеством потребителей тока. Поэтому создают системы соединений, позволяющие нагрузить один источник большим количеством потребителей. Системы при этом могут быть сколь угодно сложными и разветвленными, но в их основе лежит всего два вида соединения: последовательное и параллельное соединение проводников. Каждый вид имеет свои особенности, плюсы и минусы. Рассмотрим их оба.

Последовательное соединение проводников

Последовательное соединение проводников – это включение в электрическую цепь нескольких приборов последовательно, друг за другом. Электроприборы в данном случае можно сравнить с людьми в хороводе, а их руки, держащие друг друга – это провода, соединяющие приборы. Источник тока в данном случае будет одним из участников хоровода.

Напряжение всей цепи при последовательном соединении будет равно сумме напряжений на каждом включенном в цепь элементе. Сила тока в цепи будет одинакова в любой точке. А сумма сопротивлений всех элементов составит общее сопротивление всей цепи. Поэтому последовательное сопротивление можно выразить на бумаге следующим образом:

I=I_1=I_2=⋯=I_n  ;     U=U_1+U_2+⋯+U_n  ;     R=R_1+R_2+⋯+R_n  ,

где I — сила тока, U- напряжение, R – сопротивление,  1,2,…,n – номера элементов, включенных в цепь.

Плюсом последовательного соединения является простота сборки, а минусом – то, что если один элемент выйдет из строя, то ток пропадет во всей цепи. В такой ситуации неработающий элемент будет подобен ключу в выключенном положении. Пример из жизни неудобства такого соединения наверняка припомнят все люди постарше, которые украшали елки гирляндами из лампочек.

Если в такой гирлянде выходила из строя хотя бы одна лампочка, приходилось перебирать их все, пока не найдешь ту самую, перегоревшую. В современных гирляндах эта проблема решена. В них используют специальные диодные лампочки, в которых при перегорании сплавляются вместе контакты, и ток продолжает беспрепятственно проходить дальше.

Параллельное соединение проводников

При параллельном соединении проводников все элементы цепи подключаются к одной и той же паре точек, можно назвать их А и В. К этой же паре точек подключают источник тока. То есть получается, что все элементы подключены к одинаковому напряжению между А и В. В то же время ток как бы разделяется на все нагрузки в зависимости от сопротивления каждой из них.

Параллельное соединение можно сравнить с течением реки, на пути которой возникла небольшая возвышенность. Вода в таком случае огибает возвышенность с двух сторон, а потом вновь сливается в один поток. Получается островок посреди реки. Так вот параллельное соединение – это два отдельных русла вокруг острова. А точки А и В – это места, где разъединяется и вновь соединяется общее русло реки.

Напряжение тока в каждой отдельной ветви будет равно общему напряжению в цепи. Общий ток цепи будет складываться из токов всех отдельных ветвей. А вот общее сопротивление цепи при параллельном соединении будет меньше сопротивления тока на каждой из ветвей. Это происходит потому, что общее сечение проводника между точками А и В как бы увеличивается за счет увеличения числа параллельно подключенных нагрузок. Поэтому общее сопротивление уменьшается. Параллельное соединение описывается следующими соотношениями:

U=U_1=U_2=⋯=U_n  ;     I=I_1+I_2+⋯+I_n  ;      1/R=1/R_1 +1/R_2 +⋯+1/R_n   ,

где I — сила тока, U- напряжение, R – сопротивление,  1,2,…,n – номера элементов, включенных в цепь.

Огромным плюсом параллельного соединения является то, что при выключении одного из элементов, цепь продолжает функционировать дальше. Все остальные элементы продолжают работать. Минусом является то, что все приборы должны быть рассчитаны на одно и то же напряжение. Именно параллельным образом устанавливают розетки сети 220 В в квартирах. Такое подключение позволяет включать различные приборы в сеть совершенно независимо друг от друга, и при выходе их строя одного из них, это не влияет на работу остальных.

Нужна помощь в учебе?



Предыдущая тема: Расчёт сопротивления проводников и реостаты: формулы
Следующая тема:&nbsp&nbsp&nbspРабота и мощность тока

Последовательное и параллельное соединения источников тока

Решение:
Внутреннее сопротивление элементов

Сопротивление параллельно включенных резисторов

Общая э. д. с. элементов e0=2e Согласно закону Ома для полной цепи

15 Сопротивления резисторов R1 и R2 и э. д. с. ε1 и ε2 источников тока в схеме, изображенной на рис. 127, известны. При какой э.д.с. ε3 третьего источника ток через резистор R3 не течет?

Решение:
Выберем направления токов I1, I2 и I3 через резисторы R1, R2 и R3, указанные на рис. 363. Тогда I3=I1+I2. Разность потенциалов между точками а и b будет равна

Если

Исключая I1 находим

16 Цепь из трех одинаковых последовательно соединенных элементов с э.д.с. ε и внутренним сопротивлением r замкнута накоротко (рис. 128). Какое напряжение покажет вольтметр, подключенный к зажимам одного из элементов?

Решение:
Рассмотрим ту же схему без вольтметра (рис. 364). Из закона Ома для полной цепи находим

Из закона Ома для участка цепи между точками а и b получим

Подключение вольтметра к точкам, разность потенциалов между которыми равна нулю, ничего не может изменить в цепи. Поэтому вольтметр будет показывать напряжение, равное нулю.

17 Источник тока с э.д.с. ε0 включен в схему, параметры которой даны на рис. 129. Найти э.д.с. ε источника тока и направление его подключения к выводам а и b, при которых ток через резистор с сопротивлением R2 не идет.

Решение:
Подключим источник тока к выводам а и b и выберем направления токов, указанные на рис. 365. Для узла е имеем I=I0+I2. При обходе контуров aefb и ecdf по часовой стрелке получим

Используя условие I2 = 0, находим

Знак минус показывает, что полюсы источника тока на рис. 365 нужно поменять местами.

18 Два элемента с одинаковыми э.д.с. ε включены в цепь последовательно. Внешнее сопротивление цепи R = 5 Ом. Отношение напряжения на зажимах первого элемента к напряжению на зажимах второго элемента равно 2/3. Найти внутренние сопротивления элементов r1 и r2, если r1=2r2.

Решение:


19 Два одинаковых элемента с э.д.с. ε=1,5 В и внутренним сопротивлением r = 0,2 Ом замкнуты на резистор, сопротивление которого составляет в одном случае R1=0,2 Oм, В другом — R2 = 20 Ом. Как нужно соединить элементы (последовательно или параллельно) в первом и во втором случаях, чтобы получить наибольший ток в цепи?

Решение:
При параллельном соединении двух элементов внутреннее сопротивление и э. д.с. равны r/2 и ε при последовательном соединении они равны 2r и 2ε. Через резистор R при этом текут токи

Отсюда видно, что I2>I1, если R/2+r<R+r/2, т. е. если r1=r; следовательно, токи при параллельном и последовательном соединениях одинаковы. Во втором случае R2>r.Поэтому ток больше при последовательном соединении.

20 Два элемента с э.д.с. ε1=4В и ε2 = 2В и внутренними сопротивлениями r1 = 0,25 Ом и r2 = 0,75 Ом включены в схему, изображенную на рис. 130. Сопротивления резисторов R1 = 1 Ом и R2 = 3 Ом, емкость конденсатора С=2 мкФ. Найти заряд на конденсаторе.

Решение:


21 К батарее из двух параллельно включенных элементов с э.д.с. ε1 и ε2 и внутренними сопротивлениями r1 и r2 подключен резистор с сопротивлением R. Найти ток I, текущий через резистор R, и токи I1 и I2 в первом и втором элементах. При каких условиях токи в отдельных цепях могут быть равными нулю или изменять свое направление на обратное?

Решение:
Выберем направления токов, указанные на рис. 366. Для узла b имеем I-I1-I2=0. При обходе контуров abef и bcde по часовой стрелке получим

Из этих уравнений находим

Ток I=0 тогда, когда изменена полярность включения одного из элементов и, кроме того, выполнено условие

Ток I1=0 при

а ток I2 = 0 при

Токи I1 и I2 имеют направления, указанные на рис.366, если

Они меняют свое направление при

22 Батарея из n одинаковых аккумуляторов, соединенных в одном случае последовательно, в другом— параллельно, замыкается на резистор с сопротивлением R. При каких условиях ток, текущий через резистор, в обоих случаях будет один и тот же?

Решение:
При n(R-r) = R-r. Если R=r, то число элементов произвольно; если R№r, задача не имеет решения (n=1).

23 Батарея из n = 4 одинаковых элементов с внутренним сопротивлением r=2 Ом, соединенных в одном случае последовательно, в другом — параллельно, замыкается на резистор с сопротивлением R=10Ом. Во сколько раз показание вольтметра н одном случае отличается от показания вольтметра в другом случае? Сопротивление вольтметра велико по сравнению с R и r.

Решение:

где V1 — показание вольтметра при последовательном соединении элементов, V2-при параллельном.

24 Как изменится ток, текущий через резистор с сопротивлением R = 2 Ом, если n =10 одинаковых элементов, соединенных последовательно с этим резистором, включить параллельно ему? Э.д.с. элемента ε = 2 В, его внутреннее сопротивление r = 0,2 Ом.

Решение:


25 Батарея составлена из N=600 одинаковых элементов так, что n групп соединены последовательно и в каждой из них содержится т элементов, соединенных параллельно. Э.д.с. каждого элемента ε = 2 В, его внутреннее сопротивление r = 0,4 Ом. При каких значениях n и m батарея, будучи замкнута на внешнее сопротивление R = 0,6 Ом, отдаст во внешнюю цепь максимальную мощность? Найти при этом ток, текущий через сопротивление R.

Решение:
Общее число элементов N=nm (рис. 367). Ток во внешней цепи

где r/m— внутреннее сопротивление группы из т параллельно соединенных элементов, а nr/m — внутреннее сопротивление n групп, соединенных последовательно. Максимальная мощность отдается во внешнюю цепь при равенстве сопротивления R внутреннему сопротивлению батареи элементов nr/m, т. е.

При этом через сопротивление R течет точек I=46 А.

26 Емкость аккумулятора Qo=80А⋅ч. Найти емкость батареи из n = 3 таких аккумуляторов, включенных последовательно и параллельно.

Решение:
При последовательном соединении через все аккумуляторы батареи течет один и тот же ток, поэтому все они разрядятся в течение одного и того же времени. Следовательно, емкость батареи будет равна емкости каждого аккумулятора:
При параллельном соединении n аккумуляторов через каждый из них течет 1/n часть общего тока; поэтому при том же разрядном токе в общей цепи батареи будет разряжаться в n раз дольше, чем один аккумулятор, т. е. емкость батареи в п раз больше емкости отдельного аккумулятора:

Заметим, однако, что энергия

отдаваемая батареей в цепь, и при последовательном и при параллельном соединении n аккумуляторов в n раз больше энергии, отдаваемой одним аккумулятором. Это происходит потому, что при последовательном соединении э. д. с. батареи в n раз больше э. д. с. одного аккумулятора, а при параллельном соединении э.д.с. батареи остается той же, что и для каждого аккумулятора, но Q увеличивается в n раз.

27 Найти емкость батареи аккумуляторов, включенных по схеме, изображенной на рис.131. Емкость каждого аккумулятора Q0=64 А⋅ч.

Решение:
Каждая группа из пяти аккумуляторов, включенных последовательно, имеет емкость

Три параллельно включенные группы дают общую емкость батареи

28 Мост для измерения сопротивлений сбалансирован так, что ток через гальванометр не идет (рис. 132). Ток в правой ветви I=0,2 А. Найти напряжение V на зажимах источника тока. Сопротивления резисторов R1 = 2 Ом, R2 = 4 Ом, R3 = 1 Ом.

Решение:

29 Найти токи, протекающие в каждой ветви цепи, изображенной на рис. 133. Э.д.с. источников тока ε1 = 6,5 В и ε2 = 3,9 В. Сопротивления резисторов R1=R2=R3=R4=R5=R6=R=10 Ом.

Решение:
Составляем уравнения Кирхгофа в соответствии с направлениями токов, указанными на рис. 133: I1 + I2 — I3 = 0 для узла b;
I3 — I4 — I5 =0 для узла h; I5 — I1 — I6 = 0 для узла f: при этом

Для контура abfg (обход по часовой стрелке),

Для контура bcdh (обход против часовой стрелки) и

для контура hdef (обход по часовой стрелке). Решая эту систему уравнений с учетом, что все сопротивления одинаковы и равны R=10 Ом, получим

Отрицательные значения токов I2, I4 и I6 показывают, что при данных э. д.с. источников и сопротивлениях резисторов эти токи текут в стороны, противоположные указанным на рис. 133.

Закон ома — формулировка простыми словами, определение,

Сопротивление

Представьте, что есть труба, в которую затолкали камни. Вода, которая протекает по этой трубе, станет течь медленнее, потому что у нее появилось сопротивление. Точно также будет происходить с электрическим током.

  • Сопротивление — физическая величина, которая показывает способность проводника пропускать электрический ток. Чем выше сопротивление, тем ниже эта способность.

Теперь сделаем «каменный участок» длиннее, то есть добавим еще камней. Воде будет еще сложнее течь.

Сделаем трубу шире, оставив количество камней тем же — воде полегчает, поток увеличится.

Теперь заменим шероховатые камни, которые мы набрали на стройке, на гладкие камушки из моря. Через них проходить тоже легче, а значит сопротивление уменьшается. 2.

Знайте!

СИ — международная система единиц. «Перевести в СИ» означает перевод всех величин в метры, килограммы, секунды и другие единицы измерения без приставок. Исключение составляет килограмм с приставкой «кило».

  • Удельное сопротивление проводника — это физическая величина, которая показывает способность материала пропускать электрический ток. Это табличная величина, она зависит только от материала.

Таблица удельных сопротивлений различных материалов

Удельное сопротивление

ρ, Ом*мм2/м

Удельное сопротивление

ρ, Ом*мм2/м

Алюминий

0,028

Бронза

0,095 — 0,1

Висмут

1,2

Вольфрам

0,05

Железо

0,1

Золото

0,023

Иридий

0,0474

Константан ( сплав Ni-Cu + Mn)

0,5

Латунь

0,025 — 0,108

Магний

0,045

Манганин (сплав меди марганца и никеля — приборный)

0,43 — 0,51

Медь

0,0175

Молибден

0,059

Нейзильбер (сплав меди цинка и никеля)

0,2

Натрий

0,047

Никелин ( сплав меди и никеля)

0,42

Никель

0,087

Нихром ( сплав никеля хрома железы и марганца)

1,05 — 1,4

Олово

0,12

Платина

0. 107

Ртуть

0,94

Свинец

0,22

Серебро

0,015

Сталь

0,103 — 0,137

Титан

0,6

Хромаль

1,3 — 1,5

Цинк

0,054

Чугун

0,5-1,0

Резистор

Все реальные проводники имеют сопротивление, но его стараются сделать незначительным. В задачах вообще используют словосочетание «идеальный проводник», а значит лишают его сопротивления.

Из-за того, что проводник у нас «кругом-бегом-такой-идеальный», чаще всего за сопротивление в цепи отвечает резистор. Это устройство, которое нагружает цепь сопротивлением.

Вот так резистор изображается на схемах:


В школьном курсе физики используют Европейское обозначение, поэтому запоминаем только его. Американское обозначение можно встретить, например, в программе Micro-Cap, в которой инженеры моделируют схемы.

Вот так резистор выглядит в естественной среде обитания:


Полосочки на нем показывают его сопротивление.

На сайте компании Ekits, которая занимается продажей электронных модулей, можно выбрать цвет резистора и узнать значение его сопротивления:


Источник: сайт компании Ekits

О том, зачем дополнительно нагружать сопротивлением цепь, мы поговорим в этой же статье чуть позже.

Не сопротивляйтесь зову сердца и запишите ребенка в современную школу Skysmart. Здесь школьники решают захватывающие задачки по физике и понимают, как это пригодится в жизни.

А еще следят за прогрессом в личном кабинете, задают учителям любые — даже самые неловкие — вопросы и чувствуют себя увереннее на школьных экзаменах и контрольных. 2/м]

Закон Ома для участка цепи

С камушками в трубе все понятно, но не только же от них зависит сила, с которой поток воды идет по трубе — от насоса, которым мы эту воду качаем, тоже зависит. Чем сильнее качаем, тем больше течение. В электрической цепи функцию насоса выполняет источник тока.

Например, источником может быть гальванический элемент (привычная батарейка). Батарейка работает на основе химических реакций внутри нее. Эти реакции выделяют энергию, которая потом передается электрической цепи.

У любого источника обязательно есть полюса — «плюс» и «минус». Полюса — это его крайние положения, по сути клеммы, к которым присоединяется электрическая цепь. Собственно, ток как раз течет от «+» к «-».


У нас уже есть две величины, от которых зависит электрический ток в цепи — напряжение и сопротивление. Кажется, пора объединять их в закон.

Сила тока в участке цепи прямо пропорциональна напряжению на его концах и обратно пропорциональна его сопротивлению.

Математически его можно описать вот так:

Закон Ома для участка цепи

I = U/R

I — сила тока [A]

U — напряжение [В]

R — сопротивление [Ом]

Напряжение измеряется в Вольтах и показывает разницу между двумя точками цепи: от этой разницы зависит, насколько сильно будет течь ток — чем больше разница, тем выше напряжение и ток будет течь сильнее.

Сила тока измеряется в Амперах, а подробнее о ней вы можете прочитать в нашей статье 😇

Давайте решим несколько задач на Закон Ома для участка цепи.

Задача раз

Найти силу тока в лампочке накаливания, если торшер включили в сеть напряжением 220 В, а сопротивление нити накаливания равно 880 Ом. 2/м

Обратимся к таблице удельных сопротивлений материалов, чтобы выяснить, из какого материала сделана эта нить накаливания.

Таблица удельных сопротивлений различных материалов

Удельное сопротивление

ρ, Ом*мм2/м

Удельное сопротивление

ρ, Ом*мм2/м

Алюминий

0,028

Бронза

0,095 — 0,1

Висмут

1,2

Вольфрам

0,05

Железо

0,1

Золото

0,023

Иридий

0,0474

Константан ( сплав Ni-Cu + Mn)

0,5

Латунь

0,025 — 0,108

Магний

0,045

Манганин (сплав меди марганца и никеля — приборный)

0,43 — 0,51

Медь

0,0175

Молибден

0,059

Нейзильбер (сплав меди цинка и никеля)

0,2

Натрий

0,047

Никелин ( сплав меди и никеля)

0,42

Никель

0,087

Нихром ( сплав никеля хрома железы и марганца)

1,05 — 1,4

Олово

0,12

Платина

0. 107

Ртуть

0,94

Свинец

0,22

Серебро

0,015

Сталь

0,103 — 0,137

Титан

0,6

Хромаль

1,3 — 1,5

Цинк

0,054

Чугун

0,5-1,0

Ответ: нить накаливания сделана из константана.

Закон Ома для полной цепи

Мы разобрались с законом Ома для участка цепи. А теперь давайте узнаем, что происходит, если цепь полная: у нее есть источник, проводники, резисторы и другие элементы.

В таком случае вводится Закон Ома для полной цепи: сила тока в полной цепи равна отношению ЭДС цепи к ее полному сопротивлению.

Так, стоп. Слишком много незнакомых слов — разбираемся по-порядку.

Что такое ЭДС и откуда она берется

ЭДС расшифровывается, как электродвижущая сила. Обозначается греческой буквой ε и измеряется, как и напряжение, в Вольтах.

  • ЭДС — это сила, которая движет заряженные частицы в цепи. Она берется из источника тока. Например, из батарейки.

Химическая реакция внутри гальванического элемента (это синоним батарейки) происходит с выделением энергии в электрическую цепь. Именно эта энергия заставляет частицы двигаться по проводнику.

Зачастую напряжение и ЭДС приравнивают и говорят, что это одно и то же. Формально, это не так, но при решении задач чаще всего и правда нет разницы, так как эти величины обе измеряются в Вольтах и определяют очень похожие по сути своей процессы.

В виде формулы Закон Ома для полной цепи будет выглядеть следующим образом:

Закон Ома для полной цепи

I = ε/(R + r)

I — сила тока [A]

ε — ЭДС [В]

R — сопротивление [Ом]

r — внутреннее сопротивление источника [Ом]

Любой источник не идеален. В задачах это возможно («источник считать идеальным», вот эти вот фразочки), но в реальной жизни — точно нет. В связи с этим у источника есть внутреннее сопротивление, которое мешает протеканию тока.

Решим задачу на полную цепь.

Задачка

Найти силу тока в полной цепи, состоящей из одного резистора сопротивлением 3 Ом и источником с ЭДС равной 4 В и внутренним сопротивлением 1 Ом

Решение:

Возьмем закон Ома для полной цепи:

I = ε/(R + r)

Подставим значения:

I = 4/(3+1) = 1 A

Ответ: сила тока в цепи равна 1 А.

Когда «сопротивление бесполезно»

Электрический ток — умный и хитрый парень. Если у него есть возможность обойти резистор и пойти по идеальному проводнику без сопротивления, он это сделает. При этом с резисторами просто разных номиналов это не сработает: он не пойдет просто через меньшее сопротивление, а распределится согласно закону Ома — больше тока пойдет туда, где сопротивление меньше, и наоборот.

А вот на рисунке ниже сопротивление цепи равно нулю, потому что ток через резистор не пойдет.


Ток идет по пути наименьшего сопротивления.

Теперь давайте посмотрим на закон Ома для участка цепи еще раз.

Закон Ома для участка цепи

I = U/R

I — сила тока [A]

U — напряжение [В]

R — сопротивление [Ом]

Подставим сопротивление, равное 0. Получается, что знаменатель равен нулю, а на математике говорят, что на ноль делить нельзя. Но мы вам раскроем страшную тайну, только не говорите математикам: на ноль делить можно. Если совсем упрощать такое сложное вычисление (а именно потому что оно сложное, мы всегда говорим, что его нельзя производить), то получится бесконечность.

То есть:

I = U/0 = ∞

Такой случай называют коротким замыканием — когда величина силы тока настолько велика, что можно устремить ее к бесконечности. В таких ситуациях мы видим искру, бурю, безумие — и все ломается.

Это происходит, потому что две точки цепи имеют между собой напряжение (то есть между ними есть разница). Это как если вдоль реки неожиданно появляется водопад. Из-за этой разницы возникает искра, которую можно избежать, поставив в цепь резистор.

Именно во избежание коротких замыканий нужно дополнительное сопротивление в цепи.

Параллельное и последовательное соединение

Все это время речь шла о цепях с одним резистором. Рассмотрим, что происходит, если их больше.


Последовательное соединение

Параллельное соединение

Схема

Резисторы следуют друг за другом

Между резисторами есть два узла

Узел — это соединение трех и более проводников

Сила тока

Сила тока одинакова на всех резисторах

I = I1 = I2

Сила тока, входящего в узел, равна сумме сил токов, выходящих из него

I = I1 + I2

Напряжение

Общее напряжение цепи складывается из напряжений на каждом резисторе

U = U1 + U2

Напряжение одинаково на всех резисторах

U = U1 = U2

Сопротивление

Общее сопротивление цепи складывается из сопротивлений каждого резистора

R = R1 + R2

Общее сопротивление для бесконечного количества параллельно соединенных резисторов

1/R = 1/R1 + 1/R2 + … + 1/Rn

Общее сопротивление для двух параллельно соединенных резисторов

R = (R1 * R2)/R1 + R2

Общее сопротивление бесконечного количества параллельно соединенных одинаковых резисторов

R = R1/n

Зачем нужны эти соединения, если можно сразу взять резистор нужного номинала?

Начнем с того, что все электронные компоненты изготавливаются по ГОСТу. То есть есть определенные значения резисторов, от которых нельзя отойти при производстве. Это значит, что не всегда есть резистор нужного номинала и его нужно соорудить из других резисторов.

Параллельное соединение также используют, как «запасной аэродром»: когда на конечный результат общее сопротивление сильно не повлияет, но в случае отказа одного из резисторов, будет работать другой.

Признаемся честно: схемы, которые обычно дают в задачах (миллион параллельно соединенных резисторов, к ним еще последовательный, а к этому последовательному еще миллион параллельных) — в жизни не встречаются. Но навык расчета таких схем впоследствии упрощает подсчет схем реальных, потому что так вы невооруженным глазом отличаете последовательное соединение от параллельного.

Решим несколько задач на последовательное и параллельное соединение.

Задачка раз

Найти общее сопротивление цепи.

R1 = 1 Ом, R2 = 2 Ом, R3 = 3 Ом, R4 = 4 Ом.


Решение:

Общее сопротивление при последовательном соединении рассчитывается по формуле:

R = R1 + R2 + R3 + R4 = 1 + 2 + 3 + 4 = 10 Ом

Ответ: общее сопротивление цепи равно 10 Ом

Задачка два

Найти общее сопротивление цепи.

R1 = 4 Ом, R2 = 2 Ом


Решение:

Общее сопротивление при параллельном соединении рассчитывается по формуле:

R = (R1 * R2)/R1 + R2 = 4*2/4+2 = 4/3 = 1 ⅓ Ом

Ответ: общее сопротивление цепи равно 1 ⅓ Ом

Задачка три

Найти общее сопротивление цепи, состоящей из резистора и двух ламп.

R1 = 1 Ом, R2 = 2 Ом, R3 = 3 Ом


Решение:

Сначала обозначим, что лампы с точки зрения элемента электрической цепи не отличаются от резисторов. То есть у них тоже есть сопротивление, и они также влияют на цепь.

В данном случае соединение является смешанным. Лампы соеденены параллельно, а последовательно к ним подключен резистор.

Сначала посчитаем общее сопротивление для ламп. Общее сопротивление при параллельном соединении рассчитывается по формуле:

Rламп = (R2 * R3)/R2 + R3 = 2*3/2+3 = 6/5 = 1,2 Ом

Общее сопротивление при последовательном соединении рассчитывается по формуле:

R = R1 + Rламп = 1 + 1,2 = 2,2 Ом

Ответ: общее сопротивление цепи равно 2,2 Ом.

Наконец-то, последняя и самая сложная задача! В ней собрали все самое серьезное из этой статьи 💪.

Задачка четыре со звездочкой

К аккумулятору с ЭДС 12 В, подключена лампочка и два параллельно соединенных резистора сопротивлением каждый по 10 Ом. Известно, что ток в цепи 0,5 А, а сопротивление лампочки R/2. 2)/2R = R/2 = 10/2 = 5 Ом

И общее сопротивление цепи равно:

R = Rлампы + Rрезисторов = 5 + 5 = 10 Ом

Выразим внутреннее сопротивление источника из закона Ома для полной цепи.

I = ε/(R + r)

R + r = ε/I

r = ε/I — R

Подставим значения:

r = 12/0,5 — 10 = 14 Ом

Ответ: внутреннее сопротивление источника равно 14 Ом.

Чтобы ребенок научился решать самые сложные задачи и чувствовал себя уверенно на олимпиадах и экзаменах, запишите его на бесплатный вводный урок в Skysmart.

Профессиональные учителя физики не только научат решать задачи и подготовят к экзамену, но и объяснят, как это все устроено: легко, интерактивно и с примерами из реальной жизни современных подростков.

Учебное пособие по физике: Комбинированные схемы

Ранее в Уроке 4 упоминалось, что существует два разных способа соединения двух или более электрических устройств в цепь. Они могут быть соединены посредством последовательного или параллельного соединения. Когда все устройства в цепи соединены последовательным соединением, эта схема называется последовательной схемой. Когда все устройства в цепи соединены параллельными соединениями, тогда схема называется параллельной цепью.Третий тип схемы предполагает двойное использование последовательного и параллельного соединений в схеме; такие схемы называются составными схемами или комбинированными схемами. Схема, изображенная справа, является примером использования как последовательного, так и параллельного соединения в одной и той же цепи. В этом случае лампочки A и B подключаются параллельно, а лампочки C и D подключаются последовательно. Это пример комбинированной схемы .

При анализе комбинированных цепей критически важно иметь твердое представление о концепциях, которые относятся как к последовательным цепям, так и к параллельным цепям.Поскольку оба типа соединений используются в комбинированных схемах, концепции, связанные с обоими типами схем, применяются к соответствующим частям схемы. Основные понятия, связанные с последовательными и параллельными цепями, представлены в таблице ниже.

Цепи серии
  • Ток одинаков на всех резисторах; этот ток равен току в батарее.
  • Сумма падений напряжения на отдельных резисторах равна номинальному напряжению батареи.
  • Общее сопротивление набора резисторов равно сумме отдельных значений сопротивлений,
R к R 1 + R 2 + R 3 + …
Параллельные схемы
  • Падение напряжения одинаково на каждой параллельной ветви.
  • Сумма тока в каждой отдельной ветви равна току вне ветвей.
  • Эквивалентное или полное сопротивление набора резисторов определяется уравнением 1 / R экв = 1 / R 1 + 1 / R 2 + 1 / R 3

Каждое из вышеперечисленных понятий имеет математическое выражение. Комбинирование математических выражений вышеуказанных понятий с уравнением закона Ома (ΔV = I • R) позволяет провести полный анализ комбинированной схемы.

Анализ комбинированных схем

Основная стратегия анализа комбинированных схем включает использование значения эквивалентного сопротивления для параллельных ветвей для преобразования комбинированной схемы в последовательную. После преобразования в последовательную схему анализ можно проводить обычным образом. Ранее в Уроке 4 описывался метод определения эквивалентного параллельного сопротивления, затем общее или эквивалентное сопротивление этих ветвей равно сопротивлению одной ветви, деленному на количество ветвей.

Этот метод соответствует формуле

1 / R экв. = 1 / R 1 + 1 / R 2 + 1 / R 3 + …

, где R 1 , R 2 и R 3 — значения сопротивления отдельных резисторов, подключенных параллельно. Если два или более резистора, находящихся в параллельных ветвях, не имеют одинакового сопротивления, необходимо использовать приведенную выше формулу.Пример этого метода был представлен в предыдущем разделе Урока 4.

Применяя свое понимание эквивалентного сопротивления параллельных ветвей к комбинированной схеме, комбинированную схему можно преобразовать в последовательную. Затем понимание эквивалентного сопротивления последовательной цепи можно использовать для определения общего сопротивления цепи. Рассмотрим следующие диаграммы ниже. На схеме А представлена ​​комбинация резисторов R 2 и R 3 , размещенных в параллельных ветвях.Два параллельных резистора 4 Ом эквивалентны сопротивлению 2 Ом. Таким образом, две ветви можно заменить одним резистором с сопротивлением 2 Ом. Это показано на диаграмме B. Теперь, когда все резисторы включены последовательно, можно использовать формулу для общего сопротивления последовательных резисторов для определения общего сопротивления этой цепи: Формула для последовательного сопротивления составляет

. рандов до = 1 + 2 рандов + 3 + …

Итак, на схеме B полное сопротивление цепи составляет 10 Ом.

После определения общего сопротивления цепи анализ продолжается с использованием закона Ома и значений напряжения и сопротивления для определения значений тока в различных местах. Весь метод проиллюстрирован ниже на двух примерах.

Пример 1:

Первый пример — самый простой — резисторы, включенные параллельно, имеют одинаковое сопротивление. Цель анализа — определить ток и падение напряжения на каждом резисторе.

Как обсуждалось выше, первым шагом является упрощение схемы путем замены двух параллельных резисторов одним резистором с эквивалентным сопротивлением. Два последовательно подключенных резистора 8 Ом эквивалентны одному резистору 4 Ом. Таким образом, два резистора ответвления (R 2 и R 3 ) можно заменить одним резистором с сопротивлением 4 Ом. Этот резистор 4 Ом включен последовательно с R 1 и R 4 . Таким образом, общее сопротивление составляет

. R до = R 1 + 4 Ом + R 4 = 5 Ом + 4 Ом + 6 Ом

R общ = 15 Ом

Теперь уравнение закона Ома (ΔV = I • R) можно использовать для определения полного тока в цепи.При этом необходимо использовать общее сопротивление и общее напряжение (или напряжение батареи).

I tot = ΔV tot / R tot = (60 В) / (15 Ом)

I до = 4 А

Расчет тока 4 А представляет собой ток в месте расположения батареи. При этом резисторы R 1 и R 4 включены последовательно, а ток в последовательно соединенных резисторах везде одинаков.Таким образом,

I до = I 1 = I 4 = 4 А

Для параллельных ветвей сумма тока в каждой отдельной ветви равна току вне ветвей. Таким образом, I 2 + I 3 должно равняться 4 ампер. Существует бесконечное количество возможных значений I 2 и I 3 , которые удовлетворяют этому уравнению. Поскольку значения сопротивления равны, значения тока в этих двух резисторах также равны.Следовательно, ток в резисторах 2 и 3 равен 2 А.

I 2 = I 3 = 2 А

Теперь, когда известен ток в каждом отдельном месте резистора, можно использовать уравнение закона Ома (ΔV = I • R) для определения падения напряжения на каждом резисторе. Эти расчеты показаны ниже.

ΔV 1 = I 1 • R 1 = (4 А) • (5 Ом)
ΔV 1 = 20 В

ΔV 2 = I 2 • R 2 = (2 А) • (8 Ом)

ΔV 2 = 16 В

ΔV 3 = I 3 • R 3 = (2 А) • (8 Ом)

ΔV 3 = 16 В

ΔV 4 = I 4 • R 4 = (4 А) • (6 Ом)

ΔV 4 = 24 В

На этом анализ завершен, и его результаты представлены на диаграмме ниже.

Пример 2:

Второй пример — более сложный случай — резисторы, включенные параллельно, имеют другое сопротивление. Цель анализа та же — определить ток и падение напряжения на каждом резисторе.

Как обсуждалось выше, первым шагом является упрощение схемы путем замены двух параллельных резисторов одним резистором с эквивалентным сопротивлением.Эквивалентное сопротивление резистора 4 Ом и 12 Ом, включенного параллельно, можно определить, используя обычную формулу для эквивалентного сопротивления параллельных ветвей:

1 / R экв = 1 / R 1 + 1 / R 2 + 1 / R 3

1 / R экв = 1 / (4 Ом) + 1 / (12 Ом)

1 / R экв. = 0,333 Ом -1

R экв = 1 / (0,333 Ом -1 )

R экв = 3.00 Ом

На основании этого расчета можно сказать, что два резистора ответвления (R 2 и R 3 ) можно заменить одним резистором с сопротивлением 3 Ом. Этот резистор 3 Ом включен последовательно с R 1 и R 4 . Таким образом, общее сопротивление составляет

. R до = R 1 + 3 Ом + R 4 = 5 Ом + 3 Ом + 8 Ом

R общ = 16 Ом

Теперь уравнение закона Ома (ΔV = I • R) можно использовать для определения полного тока в цепи.При этом необходимо использовать общее сопротивление и общее напряжение (или напряжение батареи).

I tot = ΔV tot / R tot = (24 В) / (16 Ом)

I до = 1,5 А

Расчет тока 1,5 А представляет собой ток в месте расположения батареи. При этом резисторы R 1 и R 4 включены последовательно, а ток в последовательно соединенных резисторах везде одинаков.Таким образом,

I до = I 1 = I 4 = 1,5 А

Для параллельных ветвей сумма тока в каждой отдельной ветви равна току вне ветвей. Таким образом, I 2 + I 3 должно равняться 1,5 А. Существует бесконечное множество значений I 2 и I 3 , которые удовлетворяют этому уравнению. В предыдущем примере два параллельно включенных резистора имели одинаковое сопротивление; таким образом, ток распределялся поровну между двумя ветвями.В этом примере неравный ток в двух резисторах усложняет анализ. Ветвь с наименьшим сопротивлением будет иметь наибольший ток. Для определения силы тока потребуется использовать уравнение закона Ома. Но для его использования сначала необходимо знать падение напряжения на ветвях. Таким образом, направление решения в этом примере будет немного отличаться от более простого случая, проиллюстрированного в предыдущем примере.

Чтобы определить падение напряжения на параллельных ветвях, сначала необходимо определить падение напряжения на двух последовательно соединенных резисторах (R 1 и R 4 ).Уравнение закона Ома (ΔV = I • R) можно использовать для определения падения напряжения на каждом резисторе. Эти расчеты показаны ниже.

ΔV 1 = I 1 • R 1 = (1,5 А) • (5 Ом)
ΔV 1 = 7,5 В

ΔV 4 = I 4 • R 4 = (1,5 А) • (8 Ом)

ΔV 4 = 12 В

Эта схема питается от источника 24 В.Таким образом, совокупное падение напряжения заряда, проходящего по петле вокруг цепи, составляет 24 вольта. Будет падение 19,5 В (7,5 В + 12 В) в результате прохождения через два последовательно соединенных резистора (R 1 и R 4 ). Падение напряжения на ответвлениях должно составлять 4,5 В, чтобы компенсировать разницу между общим значением 24 В и падением 19,5 В на R 1 и R 4 . Таким образом,

ΔV 2 = V 3 = 4,5 В

Зная падение напряжения на параллельно соединенных резисторах (R 1 и R 4 ), можно использовать уравнение закона Ома (ΔV = I • R) для определения тока в двух ветвях.

I 2 = ΔV 2 / R 2 = (4,5 В) / (4 Ом)
I 2 = 1,125 А

I 3 = ΔV 3 / R 3 = (4,5 В) / (12 Ом)

I 3 = 0,375 A

На этом анализ завершен, и его результаты представлены на диаграмме ниже.

Разработка стратегии

Два приведенных выше примера иллюстрируют эффективную концептуально-ориентированную стратегию анализа комбинированных схем.Такой подход требовал твердого понимания концепций последовательностей и параллелей, обсуждавшихся ранее. Такие анализы часто проводятся, чтобы решить физическую проблему для указанного неизвестного. В таких ситуациях неизвестное обычно меняется от проблемы к проблеме. В одной задаче значения резистора могут быть заданы, а ток во всех ветвях неизвестен. В другой задаче могут быть указаны ток в батарее и несколько значений резистора, и неизвестная величина становится сопротивлением одного из резисторов.Очевидно, что разные проблемные ситуации потребуют небольших изменений в подходах. Тем не менее, каждый подход к решению проблем будет использовать те же принципы, что и при подходе к двум приведенным выше примерам проблем.

Начинающему студенту предлагаются следующие предложения по решению задач комбинированной схемы:

  • Если схематическая диаграмма не предоставлена, потратьте время на ее создание. Используйте условные обозначения, такие как те, что показаны в примере выше.
  • При приближении к проблеме, связанной с комбинированной схемой, найдите время, чтобы организовать себя, записав известные значения и приравняв их к символу, например, I , I 1 , R 3 , ΔV 2 и т. Д. Схема организации, использованная в двух приведенных выше примерах, является эффективной отправной точкой.
  • Знать и использовать соответствующие формулы для эквивалентного сопротивления последовательно соединенных и параллельно соединенных резисторов. Использование неправильных формул гарантирует неудачу.
  • Преобразуйте комбинированную схему в строго последовательную, заменив (на ваш взгляд) параллельную часть на один резистор, имеющий значение сопротивления, равное эквивалентному сопротивлению параллельной части.
  • Используйте уравнение закона Ома (ΔV = I • R) часто и надлежащим образом. Большинство ответов будет определено с использованием этого уравнения. При его использовании важно подставлять в уравнение соответствующие значения. Например, при вычислении I 2 важно подставить в уравнение значения ΔV 2 и R 2 .

Для дальнейшей практики анализа комбинированных схем рассмотрите возможность анализа проблем в разделе «Проверьте свое понимание» ниже.

Мы хотели бы предложить . .. Зачем просто читать об этом и когда можно с ним взаимодействовать? Взаимодействие — это именно то, что вы делаете, когда используете одну из интерактивных функций The Physics Classroom. Мы хотели бы предложить вам совместить чтение этой страницы с использованием нашего интерактивного средства построения цепей постоянного тока.Вы можете найти его в разделе Physics Interactives на нашем сайте. Построитель цепей постоянного тока предоставляет учащемуся набор для построения виртуальных цепей. Вы можете легко перетащить источники напряжения, резисторы и провода на рабочее место, а также расположить и подключить их так, как хотите. Вольтметры и амперметры позволяют измерять падение тока и напряжения. Нажатие на резистор или источник напряжения позволяет изменять сопротивление или входное напряжение. Это просто. Это весело. И это безопасно (если вы не используете его в ванне).


Проверьте свое понимание

1. Комбинированная схема показана на схеме справа. Используйте диаграмму, чтобы ответить на следующие вопросы.

а. Ток в точке A равен _____ (больше, равен, меньше) току в точке B.

г. Сила тока в точке B равна _____ (больше, равна, меньше) тока в точке E.

г. Ток в точке G равен _____ (больше, равен, меньше) ток в точке F.

г. Ток в точке E равен _____ (больше, равен, меньше) току в точке G.

e. Ток в точке B равен _____ (больше, равен, меньше) ток в точке F.

ф. Ток в точке A равен _____ (больше, равен, меньше) ток в точке L.

г. Ток в точке H равен _____ (больше, равен, меньше) ток в точке I.

2. Рассмотрим комбинированную схему на схеме справа. Используйте диаграмму, чтобы ответить на следующие вопросы. (Предположим, что падение напряжения в самих проводах пренебрежимо мало.)

а. Разность электрических потенциалов (падение напряжения) между точками B и C составляет _____ (больше, равно, меньше) разности электрических потенциалов (падение напряжения) между точками J и K.

г. Разность электрических потенциалов (падение напряжения) между точками B и K составляет _____ (больше, равно, меньше) разности электрических потенциалов (падение напряжения) между точками D и I.

г. Разность электрических потенциалов (падение напряжения) между точками E и F составляет _____ (больше, равно, меньше) разности электрических потенциалов (падение напряжения) между точками G и H.

г. Разность электрических потенциалов (падение напряжения) между точками E и F составляет _____ (больше, равно, меньше) разности электрических потенциалов (падение напряжения) между точками D и I.

e. Разность электрических потенциалов (падение напряжения) между точками J и K составляет _____ (больше, равно, меньше) разности электрических потенциалов (падение напряжения) между точками D и I.

ф. Разность электрических потенциалов между точками L и A составляет _____ (больше, равно, меньше) разности электрических потенциалов (падение напряжения) между точками B и K.


3.Используйте концепцию эквивалентного сопротивления, чтобы определить неизвестное сопротивление идентифицированного резистора, которое сделало бы схемы эквивалентными.




4. Проанализируйте следующую схему и определите значения общего сопротивления, общего тока, а также тока и падения напряжения на каждом отдельном резисторе.


5. Обращаясь к диаграмме в вопросе №4, определите …

а. … номинальная мощность резистора 4.

г. … скорость, с которой энергия потребляется резистором 3.

Разница между последовательной и параллельной цепями (со сравнительной таблицей)

Решающее различие между последовательной и параллельной цепью существует на основе ориентации компонентов в цепи. В последовательной схеме несколько компонентов соединяются каскадом, т.е. хвост одного компонента соединяется с головкой другого.

В параллельной схеме несколько компонентов соединены в ориентации голова к голове и хвост к хвосту.

Содержание: Серия против параллельной цепи

  1. Сравнительная таблица
  2. Определение
  3. Ключевые отличия
  4. Заключение


Сравнительная таблица

Основа для сравнения Последовательная цепь Параллельная цепь
Ориентация компонентов Компоненты подключаются друг за другом. Здесь компоненты соединены голова к голове и хвост к хвосту.
Ток Одинаковый ток протекает через все компоненты в цепи. Через каждый компонент цепи протекает разный ток.
Напряжение На каждом компоненте существует разная разность потенциалов (напряжение). Разность потенциалов (напряжение), существующая на различных компонентах цепи, одинакова.
Количество путей Один Несколько (зависит от количества компонентов).
Неисправность Неисправность в одном из компонентов цепи приводит к нарушению работы всей цепи. Неисправность отдельного компонента не мешает работе остальной цепи.
Устранение неисправностей Сложно. Довольно просто.
Эквивалентное сопротивление Эквивалентное сопротивление всегда больше, чем максимальное значение сопротивления в последовательном соединении. Эквивалентное сопротивление всегда меньше, чем у любого из отдельных резисторов, подключенных параллельно.

Определение последовательной цепи

В последовательной схеме компоненты в цепи подключаются один за другим или, можно сказать, каскадно. Более конкретно, мы можем сказать, что последовательная схема позволяет соединение таким образом, что хвост одного компонента напрямую соединяется с головкой другого и так далее, что соответствует двум концам батареи.

На рисунке ниже показано последовательное соединение 4 резисторов в цепи:

Поскольку мы ясно видим, что компоненты соединены каскадом в одну линию, таким образом, одинаковый ток, я буду течь через все резисторы в последовательной сети. Между тем между различными резисторами схемы существует разная разность потенциалов.

Это можно понять таким образом, что если одинаковый ток течет между всеми резисторами, то падение на каждом резисторе будет зависеть от сопротивления, предлагаемого каждым резистором в цепи.Таким образом, можно сказать, что в последовательной цепи из-за наличия единственного пути один и тот же ток течет через все компоненты. Таким образом, возникает различная разность потенциалов (напряжение) на каждом компоненте.

Определение параллельной цепи

В параллельной схеме компоненты расположены таким образом, что головки каждого компонента соединены вместе с общей точкой. Пока хвосты соединены между собой еще одной общей точкой. Тем самым образуя несколько параллельных ветвей в цепи.На рисунке показано параллельное соединение 4 резисторов в цепи:

Как мы видим здесь, параллельная схема имеет 4 ветви, и через каждую ветвь протекает разный ток. Но поскольку ветви имеют общие точки, таким образом, одинаковый потенциал существует в двух точках по отношению к двум концам потенциала батареи.

Это также можно понять так, что на каждом резисторе в цепи существует одинаковая разность потенциалов.Тогда фактический ток, протекающий через каждую ветвь, будет автоматически зависеть от сопротивления каждого резистора в цепи.

Таким образом, мы можем сказать, что из-за наличия нескольких ветвей в цепи общий ток от источника питания делится на несколько ветвей, поскольку напряжение на точках одинаково.

Ключевые различия между последовательной и параллельной схемами

  1. Компоненты в последовательной цепи расположены по единственному пути от одного конца источника питания к другому. Однако несколько компонентов в параллельной схеме расположены в множественных трактах по отношению к двум концевым выводам батареи.
  2. В последовательной цепи общий ток протекает через все компоненты цепи. В параллельной цепи через каждую параллельную ветвь цепи протекает разное количество тока.
  3. В последовательной цепи на каждом компоненте цепи присутствует различное напряжение . В то время как в параллельной цепи одинаковое напряжение присутствует на нескольких компонентах в цепи.
  4. Ошибка в одном из компонентов последовательной цепи вызывает помехи в работе всей цепи. В отличие от неисправности одного компонента в параллельной сети, не мешает функционированию другой части схемы.
  5. Обнаружение неисправности в случае последовательной цепи сложно, но довольно легко в параллельной цепи.
  6. Эквивалентное сопротивление в случае последовательной цепи всегда больше, чем наивысшее значение сопротивления в последовательном соединении. При этом эквивалентное сопротивление в параллельной цепи всегда меньше, чем любое из отдельных сопротивлений в параллельной комбинации.

Заключение

Итак, исходя из этого обсуждения, мы можем сказать, что в последовательной цепи протекающий ток остается неизменным в каждой части цепи. В параллельных цепях напряжение на двух конечных точках ветвей совпадает с подаваемым напряжением.

Последовательные и параллельные резисторы · Физика

Последовательные и параллельные резисторы · Физика
  • Нарисуйте схему с резисторами, включенными параллельно и последовательно.
  • Рассчитайте падение напряжения тока на резисторе, используя закон Ома.
  • Контраст — способ расчета общего сопротивления для резисторов, включенных последовательно и параллельно.
  • Объясните, почему полное сопротивление параллельной цепи меньше наименьшего сопротивления любого из резисторов в этой цепи.
  • Рассчитайте общее сопротивление цепи, которая содержит смесь резисторов, включенных последовательно и параллельно.

Большинство схем имеет более одного компонента, называемого резистором , который ограничивает поток заряда в цепи.Мера этого предела расхода заряда называется сопротивлением . Простейшие комбинации резисторов — это последовательное и параллельное соединение, показанное в [ссылка]. Общее сопротивление комбинации резисторов зависит как от их индивидуальных значений, так и от способа их подключения.

Резисторы серии

Когда резисторы в серии ? Резисторы включены последовательно всякий раз, когда поток заряда, называемый током , должен проходить через устройства последовательно.Например, если ток течет через человека, держащего отвертку, и попадает в Землю, тогда R1 размер 12 {R rSub {размер 8 {1}}} {}

в [ссылка] (а) может быть сопротивлением вала отвертки R2, размер 12 {R rSub {размер 8 {2}}} {}

сопротивление его ручки, R3 размер 12 {R rSub {размер 8 {3}}} {}

сопротивление тела человека и R4 размер 12 {R rSub {размер 8 {4}}} {}

сопротивление ее обуви.

[ссылка] показывает резисторы, последовательно подключенные к источнику напряжения .Кажется разумным, что полное сопротивление является суммой отдельных сопротивлений, учитывая, что ток должен проходить через каждый резистор последовательно. (Этот факт был бы преимуществом для человека, желающего избежать поражения электрическим током, который мог бы уменьшить ток, надев обувь с высоким сопротивлением на резиновой подошве. Это могло бы стать недостатком, если бы одним из сопротивлений был неисправный шнур с высоким сопротивлением. прибор, уменьшающий рабочий ток.) ​​

Чтобы убедиться, что последовательно включенные сопротивления действительно складываются, давайте рассмотрим потерю электроэнергии, называемую падением напряжения , в каждом резисторе в [ссылка].

Согласно закону Ома , падение напряжения, В, величина 12 {V} {}

, через резистор, когда через него протекает ток, рассчитывается по формуле V = размер IR 12 {V = курсив «IR»} {}

, где I размер 12 {I} {}

соответствует току в амперах (A), а размер R 12 {R} {}

— это сопротивление в Ом, размер 12 {слева (% OMEGA справа)} {}

. Другой способ думать об этом: размер V 12 {V} {}

— это напряжение, необходимое для создания тока I размера 12 {I} {}

протекает через сопротивление R размером 12 {R} {}

.

Итак, падение напряжения на R1 размером 12 {R rSub {size 8 {1}}} {}

— это V1 = IR1, размер 12 {V rSub {size 8 {1}} = ital «IR» rSub {size 8 {1}}} {}

, что для R2 размера 12 {R rSub {size 8 {2}}} {}

— это V2 = IR2, размер 12 {V rSub {size 8 {2}} = ital «IR» rSub {size 8 {2}}} {}

и R3 размера 12 {R rSub {size 8 {3}}} {}

— это V3 = IR3 размер 12 {V rSub {size 8 {3}} = ital «IR» rSub {size 8 {3}}} {}

.Сумма этих напряжений равна выходному напряжению источника; то есть

V = V1 + V2 + V3. размер 12 {V = V rSub {размер 8 {1}} + V rSub {размер 8 {2}} + V rSub {размер 8 {3}}} {}

Это уравнение основано на сохранении энергии и сохранении заряда. Электрическая потенциальная энергия может быть описана уравнением PE = qV размер 12 {ital «PE» = ital «qV»} {}

, где q размер 12 {q} {}

— это электрический заряд и V размер 12 {V} {}

— это напряжение. Таким образом, энергия, поставляемая источником, равна qV size 12 {ital «qV»} {}

, а рассеиваемая на резисторах —

qV1 + qV2 + qV3.размер 12 {ital «qV» rSub {size 8 {1}} + ital «qV» rSub {size 8 {2}} + ital «qV» rSub {size 8 {3}}} {}

Связи: законы сохранения

Вывод выражений для последовательного и параллельного сопротивления основан на законах сохранения энергии и сохранения заряда, которые утверждают, что общий заряд и полная энергия постоянны в любом процессе. Эти два закона непосредственно участвуют во всех электрических явлениях и будут многократно использоваться для объяснения как конкретных эффектов, так и общего поведения электричества.

Эти энергии должны быть равны, потому что в цепи нет другого источника и другого назначения для энергии. Таким образом, qV = qV1 + qV2 + qV3 размер 12 {ital «qV» = ital «qV» rSub {size 8 {1}} + ital «qV» rSub {size 8 {2}} + ital «qV» rSub {size 8 {3}}} {}

. Размер заряда q 12 {q} {}

отменяется, давая V = V1 + V2 + V3 размер 12 {V = V rSub {размер 8 {1}} + V rSub {размер 8 {2}} + V rSub {размер 8 {3}}} {}

, как указано. (Обратите внимание, что одинаковое количество заряда проходит через батарею и каждый резистор за заданный промежуток времени, поскольку нет емкости для хранения заряда, нет места для утечки заряда и заряд сохраняется.)

Теперь замена значений отдельных напряжений дает

V = IR1 + IR2 + IR3 = I (R1 + R2 + R3). размер 12 {V = ital «IR» rSub {size 8 {1}} + ital «IR» rSub {size 8 {2}} + ital «IR» rSub {size 8 {3}} = I \ (R rSub { размер 8 {1}} + R rSub {размер 8 {2}} + R rSub {размер 8 {3}} \)} {}

Обратите внимание, что для эквивалентного одинарного последовательного сопротивления

рупий

, у нас

V = IR.

Это означает, что полное или эквивалентное последовательное сопротивление

рупий

из трех резисторов: Rs = R1 + R2 + R3 размер 12 {R rSub {размер 8 {s}} = R rSub {размер 8 {1}} + R rSub {размер 8 {2}} + R rSub {размер 8 {3}}} {}

.

Эта логика действительна в общем для любого количества резисторов, включенных последовательно; таким образом, общее сопротивление

рупий.

последовательного подключения —

Rs = R1 + R2 + R3 + …, размер 12 {R rSub {размер 8 {s}} = R rSub {размер 8 {1}} + R rSub {размер 8 {2}} + R rSub {размер 8 { 3}} + «.» «.» «.» } {}

, как предлагается. Поскольку весь ток должен проходить через каждый резистор, он испытывает сопротивление каждого, а последовательно соединенные сопротивления просто складываются.

Расчет сопротивления, тока, падения напряжения и рассеиваемой мощности: анализ последовательной цепи

Предположим, что выходное напряжение батареи в [ссылка] равно 12.0V размер 12 {«12» «.» 0`V} {}

, а сопротивления равны R1 = 1,00 Ом размер 12 {R rSub {размер 8 {1}} = 1 «.» «00»% OMEGA} {}

, R2 = 6,00 Ом, размер 12 {R rSub {размер 8 {2}} = 6 «.» «00»% OMEGA} {}

, и R3 = 13,0 Ом, размер 12 {R rSub {size 8 {3}} = «13» «.» 0% OMEGA} {}

. а) Каково полное сопротивление? (б) Найдите ток. (c) Вычислите падение напряжения на каждом резисторе и покажите, как они складываются, чтобы равняться выходному напряжению источника. (d) Рассчитайте мощность, рассеиваемую каждым резистором.(e) Найдите выходную мощность источника и покажите, что она равна общей мощности, рассеиваемой резисторами.

Стратегия и решение для (а)

Общее сопротивление — это просто сумма отдельных сопротивлений, определяемая следующим уравнением:

Rs = R1 + R2 + R3 = 1,00 Ом + 6,00 Ом + 13,0 Ом = 20,0 Ом.

Стратегия и решение для (b)

Ток определяется по закону Ома, V = размер IR 12 {V = курсив «IR»} {}

.Ввод значения приложенного напряжения и общего сопротивления дает ток для цепи:

I = VRs = 12,0 В 20,0 Ом = 0,600 A. Размер 12 {I = {{V} больше {R rSub {size 8 {s}}}} = {{«12» «.» 0 «V»} больше {«20» «.» «0»% OMEGA}} = 0 «.» «600» «A»} {}

Стратегия и решение для (c)

Напряжение или ИК-размер 12 {ital «IR»} {}

капля — в резисторе по закону Ома. Ввод тока и значения первого сопротивления дает

. V1 = IR1 = (0.600 A) (1,0 Ом) = 0,600 В. размер 12 {V rSub {size 8 {1}} = ital «IR» rSub {size 8 {1}} = \ (0 «.» «600» «A» \ ) \ (1 «.» 0% OMEGA \) = 0 «.» «600» «V»} {}

Аналогично

V2 = IR2 = (0,600 A) (6,0 Ом) = 3,60 В, размер 12 {V rSub {size 8 {2}} = ital «IR» rSub {size 8 {2}} = \ (0 «.» «600» «A» \) \ (6 «.» 0% OMEGA \) = 3 «.» «60» «V»} {}

и

V3 = IR3 = (0,600 A) (13,0 Ом) = 7,80 В. размер 12 {V rSub {size 8 {3}} = ital «IR» rSub {size 8 {3}} = \ (0 «.» «600 «» A «\) \ (» 13 «». «0% OMEGA \) = 7″.»» 80 «» V «} {}

Обсуждение для (c)

Три ИК размера 12 {ital «IR»} {}

капли добавляются к 12,0 В для размера 12 {«12» «». 0`V} {}

, как и прогнозировалось:

V1 + V2 + V3 = (0,600 + 3,60 + 7,80) V = 12,0 В. размер 12 {V rSub {размер 8 {1}} + V rSub {размер 8 {2}} + V rSub {размер 8 {3}} = \ (0 «.» «600» +3 «.» «60» +7 «.» «80» \) «V» = «12» «.» 0 «V»} {}

Стратегия и решение для (d)

Самый простой способ рассчитать мощность в ваттах (Вт), рассеиваемую резистором в цепи постоянного тока, — это использовать закон Джоуля , P = IV размер 12 {P = курсив «IV»} {}

, где P размер 12 {P} {}

— электрическая.В этом случае через каждый резистор протекает одинаковый полный ток. Подставляя закон Ома V = IR, размер 12 {V = ital «IR»} {}

в закон Джоуля, мы получаем мощность, рассеиваемую первым резистором, как

P1 = I2R1 = (0,600 A) 2 (1,00 Ом) = 0,360 Вт. Размер 12 {P rSub {размер 8 {1}} = I rSup {размер 8 {2}} R rSub {размер 8 {1}} = \ (0 «.» «600» «A» \) rSup {size 8 {2}} \ (1 «.» «00»% OMEGA \) = 0 «.» «360» «W»} {}

Аналогично

P2 = I2R2 = (0,600 A) 2 (6,00 Ом) = 2,16 Вт, размер 12 {P rSub {размер 8 {2}} = I rSup {размер 8 {2}} R rSub {размер 8 {2}} = \ ( 0 «.»» 600 «» A «\) rSup {размер 8 {2}} \ (6». «» 00 «% OMEGA \) = 2». «» 16 «» W «} {}

и

P3 = I2R3 = (0,600 A) 2 (13,0 Ом) = 4,68 Вт. Размер 12 {P rSub {размер 8 {3}} = I rSup {размер 8 {2}} R rSub {размер 8 {3}} = \ (0 «.» «600» «A» \) rSup {size 8 {2}} \ («13» «.» 0% OMEGA \) = 4 «.». «68» «W»} {}

Обсуждение для (д)

Мощность также можно рассчитать, используя либо P = IV, размер 12 {P = ital «IV»} {}

или P = V2R размер 12 {P = {{V rSup {size 8 {2}}} больше {R}}} {}

, где V размер 12 {V} {}

— это падение напряжения на резисторе (а не полное напряжение источника).Получатся те же значения.

Стратегия и решение для (e)

Самый простой способ рассчитать выходную мощность источника — использовать P = IV размер 12 {P = ital «IV»} {}

, где V размер 12 {V} {}

— напряжение источника. Это дает

P = (0,600 A) (12,0 В) = 7,20 Вт. Размер 12 {P = \ (0 «.» «600» «A» \) \ («12» «.» 0 «V» \) = 7 » . » «20» «W»} {}

Обсуждение для (e)

Обратите внимание, что по совпадению общая мощность, рассеиваемая резисторами, также равна 7.20 Вт, столько же, сколько мощность, выдаваемая источником. То есть

P1 + P2 + P3 = (0,360 + 2,16 + 4,68) W = 7,20 W. размер 12 {P rSub {размер 8 {1}} + P rSub {размер 8 {2}} + P rSub {размер 8 {3}} = \ (0 «.» «360» +2 «.» «16» +4 «.» «68» \) «W» = 7 «.» «20» «W»} {}

Мощность — это энергия в единицу времени (ватт), поэтому для сохранения энергии требуется, чтобы выходная мощность источника была равна общей мощности, рассеиваемой резисторами.

Основные характеристики последовательно соединенных резисторов

    Сопротивления серии
  1. добавляют: Rs = R1 + R2 + R3 +…. размер 12 {R rSub {размер 8 {s}} = R rSub {размер 8 {1}} + R rSub {размер 8 {2}} + R rSub {размер 8 {3}} + «.» «.» «.» «.» } {}
  2. Одинаковый ток протекает последовательно через каждый резистор.
  3. Отдельные последовательно включенные резисторы не получают полное напряжение источника, а делят его.

Параллельные резисторы

[ссылка] показывает резисторы параллельно , подключенные к источнику напряжения. Резисторы включены параллельно, когда каждый резистор подключен непосредственно к источнику напряжения с помощью соединительных проводов с незначительным сопротивлением.Таким образом, к каждому резистору приложено полное напряжение источника.

Каждый резистор потребляет такой же ток, как если бы он один был подключен к источнику напряжения (при условии, что источник напряжения не перегружен). Например, автомобильные фары, радио и т. Д. Подключены параллельно, так что они используют полное напряжение источника и могут работать полностью независимо. То же самое и в вашем доме, или в любом другом здании. (См. [Ссылка] (b).)

Найти выражение для эквивалентного параллельного сопротивления Rp размер 12 {R rSub {size 8 {p}}} {}

, давайте рассмотрим протекающие токи и их отношение к сопротивлению.Поскольку каждый резистор в цепи имеет полное напряжение, токи, протекающие через отдельные резисторы, равны I1 = VR1 размер 12 {I rSub {размер 8 {1}} = {{V} сверх {R rSub {размер 8 {1}} }}} {}

, I2 = VR2 размер 12 {I rSub {размер 8 {2}} = {{V} больше {R rSub {size 8 {2}}}}} {}

, и I3 = VR3 размер 12 {I rSub {размер 8 {3}} = {{V} больше {R rSub {size 8 {3}}}}} {}

. Сохранение заряда подразумевает, что общий ток I размера 12 {I} {}

, произведенное источником, является суммой этих токов:

Я = I1 + I2 + I3.размер 12 {I = I rSub {размер 8 {1}} + I rSub {размер 8 {2}} + I rSub {размер 8 {3}}} {}

Подстановка выражений для отдельных токов дает

Я = VR1 + VR2 + VR3 = V1R1 + 1R2 + 1R3. размер 12 {I = {{V} больше {R rSub {размер 8 {1}}}} + {{V} больше {R rSub {размер 8 {2}}}} + {{V} больше {R rSub { размер 8 {3}}}} = V слева ({{1} больше {R rSub {размер 8 {1}}}} + {{1} больше {R rSub {размер 8 {2}}}} + {{ 1} больше {R rSub {size 8 {3}}}} вправо)} {}

Обратите внимание, что закон Ома для эквивалентного одиночного сопротивления дает

I = VRp = V1Rp.размер 12 {I = {{V} над {R rSub {размер 8 {p}}}} = V слева ({{1} над {R rSub {размер 8 {p}}}} справа)} {}

Члены в круглых скобках в последних двух уравнениях должны быть равны. Обобщая на любое количество резисторов, общее сопротивление Rp размером 12 {R rSub {size 8 {p}}} {}

параллельного соединения связано с отдельными сопротивлениями на

1Rp = 1R1 + 1R2 + 1R.3 + …. размер 12 {{{1} больше {R rSub {size 8 {p}}}} = {{1} больше {R rSub {размер 8 {1}} }} + {{1} больше {R rSub {размер 8 {2}}}} + {{1} больше {R rSub {размер 8 {«.»3}}}} +». «». «». «». «} {}

Это соотношение приводит к общему сопротивлению Rp размером 12 {R rSub {size 8 {p}}} {}

, что меньше наименьшего из отдельных сопротивлений. (Это видно в следующем примере.) При параллельном подключении резисторов от источника течет больше тока, чем протекает по любому из них по отдельности, поэтому общее сопротивление ниже.

Расчет сопротивления, тока, рассеиваемой мощности и выходной мощности: анализ параллельной цепи

Пусть выходное напряжение батареи и сопротивления в параллельном соединении в [link] будут такими же, как и в ранее рассмотренном последовательном соединении: V = 12.0 В, размер 12 {V = «12» «.» 0 «V»} {}

, R1 = 1,00 Ом, размер 12 {R rSub {размер 8 {1}} = 1 «.» «00»% OMEGA} {}

, R2 = 6,00 Ом, размер 12 {R rSub {размер 8 {2}} = 6 «.» «00»% OMEGA} {}

, и R3 = 13,0 Ом, размер 12 {R rSub {size 8 {3}} = «13» «.» 0% OMEGA} {}

. а) Каково полное сопротивление? (б) Найдите полный ток. (c) Рассчитайте токи в каждом резисторе и покажите, как они складываются, чтобы равняться общему выходному току источника. (d) Рассчитайте мощность, рассеиваемую каждым резистором.(e) Найдите выходную мощность источника и покажите, что она равна общей мощности, рассеиваемой резисторами.

Стратегия и решение для (а)

Общее сопротивление для параллельной комбинации резисторов находится с помощью следующего уравнения. Ввод известных значений дает

1Rp = 1R1 + 1R2 + 1R3 = 11,00 Ом + 16,00 Ом + 113,0 Ом. размер 12 {{{1} больше {R rSub {размер 8 {p}}}} = {{1} больше {R rSub {размер 8 {1}}}} + {{1} больше {R rSub {размер 8 {2}}}} + {{1} больше {R rSub {size 8 {3}}}} = {{1} больше {1 «.»» 00 «% OMEGA}} + {{1} более {6». «» 00 «% OMEGA}} + {{1} более {» 13 «». «0% OMEGA}}} {}

Таким образом,

1Rp = 1,00 Ом + 0,1667 Ом + 0,07692 Ом = 1,2436 Ом. размер 12 {{{1} больше {R rSub {size 8 {p}}}} = {{1 «.» «00»} больше {% OMEGA}} + {{0 «.» «167»} больше {% OMEGA}} + {{0 «.» «0769»} более {% OMEGA}} = {{1 «.» «244»} больше {% OMEGA}}} {}

(Обратите внимание, что в этих расчетах каждый промежуточный ответ отображается с дополнительной цифрой.)

Мы должны перевернуть это, чтобы найти полное сопротивление Rp размер 12 {R rSub {размер 8 {p}}} {}

.Это дает

Rp = 11,2436 Ом = 0,8041 Ом. размер 12 {R rSub {размер 8 {p}} = {{1} более {1 «.» «2436»}}% OMEGA = 0 «.» «8041»% OMEGA} {}

Общее сопротивление с правильным количеством значащих цифр Rp = 0,804 Ом. размер 12 {R rSub {размер 8 {p}} = 0 «.» «804»% OMEGA} {}

Обсуждение для (а)

Rp

, как и предполагалось, меньше минимального индивидуального сопротивления.

Стратегия и решение для (b)

Полный ток можно найти из закона Ома, заменив Rp размер 12 {R rSub {размер 8 {p}}} {}

для общего сопротивления.Это дает

I = VRp = 12,0 V0,8041 Ом = 14,92 A. размер 12 {I = {{V} over {R rSub {size 8 {p}}}} = {{«12» «.» 0 «V»} больше {0 «.» «804»% OMEGA}} = «14» «.» «92» «A»} {}

Обсуждение для (б)

Текущий размер I 12 {I} {}

для каждого устройства намного больше, чем для тех же устройств, подключенных последовательно (см. Предыдущий пример). Схема с параллельным соединением имеет меньшее общее сопротивление, чем резисторы, включенные последовательно.

Стратегия и решение для (c)

Отдельные токи легко вычислить по закону Ома, поскольку каждый резистор получает полное напряжение.Таким образом,

I1 = VR1 = 12,0 В, 1,00 Ом = 12,0 А. размер 12 {I rSub {размер 8 {1}} = {{V} больше {R rSub {размер 8 {1}}}} = {{«12″ » . » 0 «V»} больше {1 «.» «00»% OMEGA}} = «12» «.» 0 «А»} {}

Аналогично

I2 = VR2 = 12,0 В 6,00 Ом = 2,00 А размер 12 {I rSub {размер 8 {2}} = {{V} больше {R rSub {размер 8 {2}}}} = {{«12» «. » 0 «V»} больше {6 «.» «00»% OMEGA}} = 2 «.» «00» «A»} {}

и

I3 = VR3 = 12,0 В 13,0 Ом = 0,92 А. размер 12 {I rSub {размер 8 {3}} = {{V} больше {R rSub {размер 8 {3}}}} = {{«12″ » .»0″ V «} больше {» 13 «». «» 0 «% OMEGA}} = 0». «» 92 «» A «} {}

Обсуждение для (c)

Общий ток складывается из отдельных токов:

I1 + I2 + I3 = 14,92 A. размер 12 {I rSub {размер 8 {1}} + I rSub {размер 8 {2}} + I rSub {размер 8 {3}} = «14» «.» «92» «A»} {}

Это соответствует сохранению заряда.

Стратегия и решение для (d)

Мощность, рассеиваемая каждым резистором, может быть найдена с помощью любого из уравнений, связывающих мощность с током, напряжением и сопротивлением, поскольку все три известны.Давайте использовать P = V2R размер 12 {P = {{V rSup {size 8 {2}}} вместо {R}}} {}

, поскольку каждый резистор получает полное напряжение. Таким образом,

P1 = V2R1 = (12,0 В) 21,00 Ом = 144 Вт. Размер 12 {P rSub {размер 8 {1}} = {{V rSup {размер 8 {2}}} больше {R rSub {размер 8 {1}}) }} = {{\ («12» «.» 0 «V» \) rSup {size 8 {2}}} больше {1 «.» «00»% OMEGA}} = «144» «W»} {}

Аналогично

P2 = V2R2 = (12,0 В) 26,00 Ом = 24,0 Вт размер 12 {P rSub {размер 8 {2}} = {{V rSup {размер 8 {2}}} больше {R rSub {размер 8 {2}}} } = {{\ («12» «.»0» V «\) rSup {size 8 {2}}} больше {6». «» 00 «% OMEGA}} =» 24 «». «0» W «} {}

и

P3 = V2R3 = (12,0 В) 213,0 Ом = 11,1 Вт. Размер 12 {P rSub {размер 8 {3}} = {{V rSup {размер 8 {2}}} больше {R rSub {размер 8 {3}}) }} = {{\ («12» «.» 0 «V» \) rSup {size 8 {2}}} больше {«13» «.» «0»% OMEGA}} = «11» «.» 1 «Вт»} {}

Обсуждение для (д)

Мощность, рассеиваемая каждым резистором при параллельном подключении, значительно выше, чем при последовательном подключении к тому же источнику напряжения.

Стратегия и решение для (e)

Общую мощность также можно рассчитать несколькими способами. Выбираем P = IV размер 12 {P = ital «IV»} {}

, а ввод полного тока дает

P = IV = (14,92 A) (12,0 В) = 179 Вт. Размер 12 {P = ital «IV» = \ («14» «.» «92» «A» \) \ («12» «.». » 0 «V» \) = «179» «.» 1 «Вт»} {}

Обсуждение для (e)

Суммарная мощность, рассеиваемая резисторами, также 179 Вт:

P1 + P2 + P3 = 144 Вт + 24.0 Вт + 11,1 Вт = 179 Вт. размер 12 {P rSub {размер 8 {1}} + P rSub {размер 8 {2}} + P rSub {размер 8 {3}} = «144» «W» + «24» «.» 0 «W» + «11» «.» 1 «W» = «179» «W»} {}

Это соответствует закону сохранения энергии.

Общее обсуждение

Обратите внимание, что как токи, так и мощность при параллельном подключении больше, чем для тех же устройств, подключенных последовательно.

Основные характеристики параллельных резисторов

  1. Параллельное сопротивление находится от 1Rp = 1R1 + 1R2 + 1R3 +… размер 12 {{{1} больше {R rSub {размер 8 {p}}}} = {{1} больше {R rSub {размер 8 {1}}}} + {{1} больше {R rSub { размер 8 {2}}}} + {{1} больше {R rSub {size 8 {3}}}} + «.» «.» «.» } {}

    , и это меньше любого отдельного сопротивления в комбинации.

  2. Каждый резистор, включенный параллельно, имеет то же полное напряжение, что и источник. (В системах распределения электроэнергии чаще всего используются параллельные соединения для питания бесчисленных устройств, обслуживаемых одним и тем же напряжением, и для того, чтобы они могли работать независимо.)
  3. Не каждый параллельный резистор получает полный ток; они делят это.

Сочетания последовательного и параллельного

Более сложные соединения резисторов иногда представляют собой просто комбинации последовательного и параллельного. Они часто встречаются, особенно если учитывать сопротивление провода. В этом случае сопротивление провода включено последовательно с другими сопротивлениями, включенными параллельно.

Комбинации последовательного и параллельного подключения можно свести к одному эквивалентному сопротивлению, используя метод, показанный в [ссылка].Различные части идентифицируются как последовательные или параллельные, уменьшаются до их эквивалентов и далее уменьшаются до тех пор, пока не останется единственное сопротивление. Процесс занимает больше времени, чем труден.

Простейшая комбинация последовательного и параллельного сопротивления, показанная в [ссылка], также является наиболее поучительной, поскольку она используется во многих приложениях. Например, размер R1 12 {R rSub {размер 8 {1}}} {}

может быть сопротивлением проводов от автомобильного аккумулятора к его электрическим устройствам, которые соединены параллельно.R2 размер 12 {R rSub {размер 8 {1}}} {}

и R3 размер 12 {R rSub {размер 8 {1}}} {}

мог быть стартером и светом салона. Ранее мы предполагали, что сопротивление провода незначительно, но, когда это не так, оно имеет важные последствия, как показывает следующий пример.

Расчет сопротивления, Размер ИК 12 {ital «IR»} {} Падение, ток и рассеяние мощности: объединение последовательных и параллельных цепей

[ссылка] показывает резисторы из двух предыдущих примеров, подключенные другим способом — комбинацией последовательного и параллельного.Мы можем рассмотреть R1 размера 12 {R rSub {size 8 {1}}} {}

— сопротивление проводов, ведущих к R2 размером 12 {R rSub {размер 8 {2}}} {}

и R3 размер 12 {R rSub {размер 8 {3}}} {}

. (а) Найдите полное сопротивление. (б) Что такое ИК-размер 12 {итал. «ИК»} {}

капля R1 размер 12 {R rSub {размер 8 {1}}} {}

? (c) Найдите текущий размер I2 12 {I rSub {size 8 {2}}} {}

С

по R2 размер 12 {R rSub {размер 8 {2}}} {}

.(d) Какая мощность рассеивается R2 размером 12 {R rSub {размер 8 {2}}} {}

?

Стратегия и решение для (а)

Чтобы найти полное сопротивление, отметим, что R2 размер 12 {R rSub {размер 8 {2}}} {}

и R3 размер 12 {R rSub {размер 8 {3}}} {}

параллельно и их комбинация Rp размер 12 {R rSub {size 8 {p}}} {}

входит в серию с R1 размером 12 {R rSub {размер 8 {1}}} {}

.Таким образом, полное (эквивалентное) сопротивление этой комбинации составляет

. Rtot = R1 + Rp. размер 12 {R rSub {размер 8 {«tot»}} = R rSub {размер 8 {1}} + R rSub {размер 8 {p}}} {}

Сначала находим Rp size 12 {R rSub {size 8 {p}}} {}

, используя уравнение для параллельных резисторов и вводя известные значения:

1Rp = 1R2 + 1R3 = 16,00 Ом + 113,0 Ом = 0,2436 Ом. размер 12 {{{1} больше {R rSub {размер 8 {p}}}} = {{1} больше {R rSub {размер 8 {2}}}} + {{1} больше {R rSub {размер 8 {3}}}} = {{1} более {6 «.»» 00 «% OMEGA}} + {{1} более {» 13 «». «0% OMEGA}} = {{0». «» 2436 «} более {% OMEGA}}} {}

Инвертирование дает

Rp = 10,2436 Ом = 4,11 Ом. размер 12 {R rSub {размер 8 {p}} = {{1} более {0 «.» «2436»}}% OMEGA = 4 «.» «11»% OMEGA} {}

Таким образом, общее сопротивление равно

. Rtot = R1 + Rp = 1,00 Ом + 4,11 Ом = 5,11 Ом. размер 12 {R rSub {размер 8 {«tot»}} = R rSub {размер 8 {1}} + R rSub {размер 8 {p}} = 1 «.» «00»% OMEGA +4 «.» «11»% OMEGA = 5 «.» «11»% OMEGA} {}

Обсуждение для (а)

Общее сопротивление этой комбинации является промежуточным между значениями чистой серии и чистой параллели (20.0 Ом

и 0,804 Ом

соответственно), найденных для тех же резисторов в двух предыдущих примерах.

Стратегия и решение для (b)

Чтобы найти ИК-размер 12 {ital «IR»} {}

капля R1 размер 12 {R rSub {размер 8 {1}}} {}

, отметим, что полный ток I размера 12 {I} {}

проходит через R1 размером 12 {R rSub {size 8 {1}}} {}

. Таким образом, его размер IR 12 {ital «IR»} {}

дроп — это

V1 = IR1.размер 12 {V rSub {размер 8 {1}} = ital «IR» rSub {size 8 {1}}} {}

Мы должны найти I размер 12 {I} {}

, прежде чем мы сможем вычислить размер V1 12 {V rSub {size 8 {1}}} {}

. Суммарный ток I размера 12 {I} {}

находится с помощью закона Ома для схемы. То есть

I = VRtot = 12,0 В 5,11 Ом = 2,35 А. размер 12 {I = {{V} больше {R rSub {size 8 {«tot»}}}} = {{«12» «.» 0 «V»} больше {5 «.» «11»% OMEGA}} = 2 «.» «35» «A»} {}

Вводя это в выражение выше, мы получаем

V1 = IR1 = (2.35 A) (1,00 Ом) = 2,35 В. размер 12 {V rSub {size 8 {1}} = ital «IR» rSub {size 8 {1}} = \ (2 «.» «35» «A» \ ) \ (1 «.» «00»% OMEGA \) = 2 «.» «35» «V»} {}

Обсуждение для (б)

Напряжение, приложенное к R2 размером 12 {R rSub {размер 8 {2}}} {}

и R3 размер 12 {R rSub {размер 8 {3}}} {}

меньше общего напряжения на величину V1 размер 12 {V rSub {размер 8 {1}}} {}

. Большое сопротивление провода может существенно повлиять на работу устройств, представленных R2 размером 12 {R rSub {размер 8 {2}}} {}

и R3 размер 12 {R rSub {размер 8 {3}}} {}

.

Стратегия и решение для (c)

Чтобы найти ток через R2 размер 12 {R rSub {размер 8 {2}}} {}

, мы должны сначала найти приложенное к нему напряжение. Мы называем это напряжение Vp размером 12 {V rSub {size 8 {p}}} {}

, потому что он применяется к параллельной комбинации резисторов. Напряжение, приложенное к обоим R2 типоразмера 12 {R rSub {размер 8 {2}}} {}

и R3 размер 12 {R rSub {размер 8 {3}}} {}

уменьшается на величину V1, размер 12 {V rSub {size 8 {1}}} {}

, так что это

Vp = V − V1 = 12.0 В − 2,35 В = 9,65 В. Размер 12 {V rSub {размер 8 {p}} = V — V rSub {размер 8 {1}} = «12» «.» 0 «В» — 2 «.» «35» «V» = 9 «.» «65» «V»} {}

Теперь текущий размер I2 12 {I rSub {size 8 {2}}} {}

через сопротивление R2 размер 12 {R rSub {размер 8 {2}}} {}

находится по закону Ома:

I2 = VpR2 = 9,65 V6,00 Ом = 1,61 A. размер 12 {I rSub {размер 8 {2}} = {{V rSub {размер 8 {p}}} больше {R rSub {размер 8 {2}}} } = {{9 «.» «65 В»} более {6 «.» «00»% OMEGA}} = 1 «.» «61» «A»} {}

Обсуждение для (c)

Ток меньше 2.00 A, который прошел через R2 размера 12 {R rSub {размер 8 {2}}} {}

, когда он был подключен параллельно к батарее в предыдущем примере параллельной схемы.

Стратегия и решение для (d)

Мощность, рассеиваемая R2 размером 12 {R rSub {размер 8 {2}}} {}

выдается

P2 = (I2) 2R2 = (1,61 A) 2 (6,00 Ом) = 15,5 Вт. Размер 12 {P rSub {размер 8 {2}} = \ (I rSub {размер 8 {2}} \) rSup {размер 8 {2}} R rSub {размер 8 {2}} = \ (1 «.» «61» «A» \) rSup {размер 8 {2}} \ (6 «.»» 00 «% OMEGA \) =» 15 «». «5» W «} {}

Обсуждение для (д)

Мощность меньше 24,0 Вт, рассеиваемых этим резистором при параллельном подключении к источнику 12,0 В.

Практическое применение

Одним из следствий этого последнего примера является то, что сопротивление в проводах снижает ток и мощность, подаваемую на резистор. Если сопротивление провода относительно велико, как в изношенном (или очень длинном) удлинителе, то эти потери могут быть значительными. Если потребляется большой ток, размер IR 12 {ital «IR»} {}

Падение

в проводах также может быть значительным.

Например, когда вы роетесь в холодильнике и включается мотор, свет холодильника на мгновение гаснет. Точно так же вы можете увидеть тусклый свет в салоне, когда вы запускаете двигатель вашего автомобиля (хотя это может быть связано с сопротивлением внутри самой батареи).

Что происходит в этих сильноточных ситуациях, показано в [ссылка]. Устройство представлено размером R3 12 {R rSub {размер 8 {3}}} {}

имеет очень низкое сопротивление, поэтому при включении протекает большой ток.Этот увеличенный ток приводит к увеличению размера IR 12 {ital «IR»} {}

Падение

в проводах, представленное R1 размером 12 {R rSub {размер 8 {1}}} {}

, уменьшая напряжение на лампочке (это R2 размер 12 {R rSub {размер 8 {2}}} {}

), который затем заметно тускнеет.

Проверьте свое понимание

Можно ли любую произвольную комбинацию резисторов разбить на последовательную и параллельную? Посмотрите, сможете ли вы нарисовать принципиальную схему резисторов, которые нельзя разбить на комбинации последовательно и параллельно.

Нет, есть много способов подключения резисторов, которые не являются комбинациями последовательного и параллельного, включая петли и переходы. В таких случаях правила Кирхгофа, которые будут включены в Правила Кирхгофа, позволят вам проанализировать схему.

Стратегии решения проблем для последовательных и параллельных резисторов

  1. Нарисуйте четкую принципиальную схему, обозначив все резисторы и источники напряжения. Этот шаг включает список известных проблем, поскольку они отмечены на вашей принципиальной схеме.
  2. Определите, что именно необходимо определить в проблеме (определите неизвестные). Письменный список полезен.
  3. Определите, включены ли резисторы последовательно, параллельно или в комбинации последовательно и параллельно. Изучите принципиальную схему, чтобы сделать эту оценку. Резисторы включены последовательно, если через них должен последовательно проходить один и тот же ток.
  4. Используйте соответствующий список основных функций для последовательных или параллельных соединений, чтобы найти неизвестные.Есть один список для серий, а другой — для параллелей. Если ваша проблема представляет собой комбинацию последовательного и параллельного соединения, уменьшайте ее поэтапно, рассматривая отдельные группы последовательных или параллельных соединений, как это сделано в этом модуле и примерах. Особое примечание: при нахождении Rp, размер 12 {R «» lSub {size 8 {p}}} {}

    , с обратным следует обращаться осторожно.

  5. Проверьте, являются ли ответы разумными и последовательными. Единицы и числовые результаты должны быть разумными.Общее последовательное сопротивление должно быть больше, а общее параллельное сопротивление, например, должно быть меньше. Мощность должна быть больше для одних и тех же устройств, подключенных параллельно, по сравнению с последовательными и т. Д.

Сводка раздела

  • Общее сопротивление электрической цепи с последовательно включенными резисторами является суммой отдельных сопротивлений: Rs = R1 + R2 + R3 + …. размер 12 {R rSub {размер 8 {s}} = R rSub {размер 8 {1}} + R rSub {размер 8 {2}} + R rSub {размер 8 { 3}} + «.»». «». «». «} {}
  • Каждый резистор в последовательной цепи имеет одинаковое количество тока, протекающего через него.
  • Падение напряжения или рассеиваемая мощность на каждом отдельном резисторе в серии различаются, и их общая сумма складывается с входом источника питания.
  • Общее сопротивление электрической цепи с резисторами, подключенными параллельно, меньше наименьшего сопротивления любого из компонентов и может быть определено по формуле: 1Rp = 1R1 + 1R2 + 1R3 +…. размер 12 {{{1} больше {R rSub {размер 8 {p}}}} = {{1} больше {R rSub {размер 8 {1}}}} + {{1} больше {R rSub {размер 8 {2}}}} + {{1} больше {R rSub {size 8 {3}}}} + «.» «.» «.» «.» } {}
  • Каждый резистор в параллельной цепи имеет одинаковое полное напряжение источника.
  • Ток, протекающий через каждый резистор в параллельной цепи, различается в зависимости от сопротивления.
  • Если более сложное соединение резисторов представляет собой комбинацию последовательного и параллельного резисторов, оно может быть уменьшено до одного эквивалентного сопротивления, идентифицируя его различные части как последовательные или параллельные, уменьшая каждую до своего эквивалента и продолжая до тех пор, пока в конечном итоге не будет достигнуто единое сопротивление. .

Концептуальные вопросы

Переключатель имеет переменное сопротивление, близкое к нулю в замкнутом состоянии и очень большое в разомкнутом, и он подключается последовательно с устройством, которым он управляет. Объясните влияние переключателя в [ссылка] на ток в разомкнутом и замкнутом состоянии.

Какое напряжение на разомкнутом переключателе в [ссылка]?

Есть напряжение на разомкнутом переключателе, например, в [ссылка]. Почему же тогда мощность, рассеиваемая разомкнутым переключателем, мала?

Почему мощность, рассеиваемая замкнутым переключателем, например в [link], мала?

Студент в физической лаборатории по ошибке подключил электрическую лампочку, батарею и выключатель, как показано на [ссылка].Объясните, почему лампочка горит, когда переключатель разомкнут, и гаснет, когда переключатель замкнут. (Не пытайтесь — батарея сильно разряжается!)

Зная, что сила удара зависит от величины тока, протекающего через ваше тело, вы бы предпочли, чтобы он был включен последовательно или параллельно с сопротивлением, таким как нагревательный элемент тостера, если он поражен им? Объяснять.

Были бы ваши фары тусклыми при запуске двигателя автомобиля, если бы провода в вашем автомобиле были сверхпроводниками? (Не пренебрегайте внутренним сопротивлением батареи.) Объяснять.

Некоторые гирлянды праздничных огней соединены последовательно для экономии затрат на проводку. В старой версии использовались лампочки, которые при перегорании прерывали электрическое соединение, как открытый выключатель. Если одна такая лампочка перегорит, что случится с остальными? Если такая цепочка работает от 120 В и имеет 40 одинаковых лампочек, каково нормальное рабочее напряжение каждой? В более новых версиях используются лампы, которые при перегорании замыкаются накоротко, как замкнутый выключатель. Если одна такая лампочка перегорит, что случится с остальными? Если такая цепочка работает от 120 В и в ней осталось 39 идентичных лампочек, каково тогда рабочее напряжение каждой?

Если две бытовые лампочки мощностью 60 Вт и 100 Вт подключить последовательно к бытовой электросети, какая из них будет ярче? Объяснять.

Предположим, вы проводите физическую лабораторию, в которой вас просят вставить резистор в цепь, но все прилагаемые резисторы имеют большее сопротивление, чем запрошенное значение. Как бы вы соединили доступные сопротивления, чтобы попытаться получить меньшее запрошенное значение?

Перед Второй мировой войной некоторые радиостанции получали питание через «шнур сопротивления», который имел значительное сопротивление. Такой резистивный шнур снижает напряжение до желаемого уровня для ламп радиоприемника и т.п., и это экономит расходы на трансформатор.Объясните, почему шнуры сопротивления нагреваются и тратят энергию при включенном радио.

У некоторых лампочек есть три уровня мощности (не включая ноль), получаемые от нескольких нитей накала, которые индивидуально переключаются и соединяются параллельно. Какое минимальное количество нитей необходимо для трех режимов мощности?

Проблемные упражнения

Примечание. Можно считать, что данные, взятые из цифр, имеют точность до трех значащих цифр.

(a) Каково сопротивление десяти 275-омных размеров 12 {«275» -% OMEGA} {}

резисторов соединены последовательно? (б) Параллельно?

(а) 2.75 кОм размер 12 {2 «.» «75» «k»% OMEGA} {}

(b) 27,5 Ом, размер 12 {«27» «.» 5% ОМЕГА} {}

(a) Каково сопротивление 1,00 × 102 Ом

, 2,50 кОм

и 4,00 кОм размером 12 {4 «.» «00» «- k»% OMEGA} {}

резистор соединен последовательно? (б) Параллельно?

Какое наибольшее и наименьшее сопротивление можно получить, подключив 36,0 Ом

, размер 50,0 Ом 12 {«50» «.» 0-% OMEGA} {}

и размер 700 Ом 12 {«700» -% OMEGA} {}

резистор вместе?

(a) 786 Ом, размер 12 {«786″% OMEGA} {}

(б) 20.3Ω размер 12 {«20» «.» 3% ОМЕГА} {}

Тостер на 1800 Вт, электрическая сковорода на 1400 Вт и лампа на 75 Вт подключены к одной розетке в цепи 15 А, 120 В. (Три устройства работают параллельно, если подключены к одной розетке.) а) Какой ток потребляет каждое устройство? (b) Перегорит ли эта комбинация предохранитель на 15 А?

Фара мощностью 30,0 Вт и стартер мощностью 2,40 кВт обычно подключаются параллельно в систему на 12,0 В. Какую мощность потребляли бы одна фара и стартер при последовательном подключении к 12.0-вольтовый аккумулятор? (Не обращайте внимания на любое другое сопротивление в цепи и любое изменение сопротивления в двух устройствах.)

29,6 Вт, размер 12 {«29» «.» 6 «Вт»} {}

(a) Учитывая батарею на 48,0 В и 24,0 Ом, размер 12 {«24» «». » 0-% OMEGA} {}

и 96,0 Ом, размер 12 {«96» «». 0-% OMEGA} {}

резисторов, найдите ток и мощность для каждого при последовательном соединении. (b) Повторите, когда сопротивления включены параллельно.

Ссылаясь на пример комбинирования последовательных и параллельных цепей и [ссылка], вычислите размер I3 12 {I rSub {размер 8 {3}}} {}

двумя следующими способами: (a) из известных значений I размера 12 {I} {}

и I2 размер 12 {I rSub {размер 8 {2}}} {}

; (б) используя закон Ома для R3 размера 12 {R rSub {size 8 {3}}} {}

.В обеих частях явно показано, как вы выполняете шаги, указанные в Стратегиях решения проблем для последовательных и параллельных резисторов.

Ссылаясь на [ссылку]: (a) Рассчитайте размер P3 12 {P rSub {размер 8 {3}}} {}

и обратите внимание на его сравнение с размером P3 12 {P rSub {size 8 {3}}} {}

обнаружил в первых двух примерах проблем в этом модуле. (b) Найдите полную мощность, отдаваемую источником, и сравните ее с суммой мощностей, рассеиваемых резисторами.

См. [Ссылка] и обсуждение затемнения света при включении тяжелого прибора. (a) Если источник напряжения составляет 120 В, сопротивление провода составляет 0,400 Ом размером 12 {0 «.» «800»% OMEGA} {}

, а номинальная мощность лампы составляет 75,0 Вт. Какую мощность рассеивает лампа, если при включении двигателя по проводам проходит всего 15,0 А? Предположите незначительное изменение сопротивления лампы. б) Какая мощность потребляет двигатель?

ЛЭП 240 кВ по 5.00 × 102A

подвешен на заземленных металлических опорах с помощью керамических изоляторов, каждая из которых имеет размер 1,00 × 109 Ом 12 {1 «». «00» ´ «10» rSup {размер 8 {9}} -% OMEGA} {}

сопротивление. [ссылка на сайт]. а) Какое сопротивление относительно земли у 100 изоляторов? (b) Рассчитайте мощность, рассеиваемую 100 из них. (c) Какая доля мощности, переносимой линией, составляет это? Ясно покажите, как вы выполняете шаги, указанные в Стратегиях решения проблем для последовательных и параллельных резисторов.

Покажите, что если два резистора R1 размером 12 {R rSub {размер 8 {1}}} {}

и R2 размер 12 {R rSub {размер 8 {2}}} {}

объединены, и один намного больше другого (R1 >> R2 размер 12 {R rSub {размер 8 {1}} «>>» R rSub {размер 8 {2}}} {}

): (a) Их последовательное сопротивление почти равно большему сопротивлению R1 размера 12 {R rSub {size 8 {1}}} {}

.(b) Их параллельное сопротивление почти равно меньшему сопротивлению R2 размера 12 {R rSub {размер 8 {2}}} {}

.

(a) Rs = R1 + R2⇒Rs≈R1R1 >> R2alignl {стек {размер 12 {R rSub {размер 8 {s}} = R rSub {размер 8 {1}} + R rSub {размер 8 {2}) }} {} # drarrow R rSub {размер 8 {s}} »R rSub {размер 8 {1}} слева (R rSub {размер 8 {1}}» >> «R rSub {размер 8 {2}} справа ) {}}} {}

(b) 1Rp = 1R1 + 1R2 = R1 + R2R1R2 размер 12 {{{1} больше {R rSub {размер 8 {p}}}} = {{1} больше {R rSub {размер 8 {1}}} } + {{1} больше {R rSub {размер 8 {2}}}} = {{R rSub {размер 8 {1}} + R rSub {размер 8 {2}}} больше {R rSub {размер 8 { 1}} R rSub {размер 8 {2}}}}} {}

,

, так что

Rp = R1R2R1 + R2≈R1R2R1 = R2R1 >> R2.выровняйте {стек {размер 12 {R rSub {размер 8 {p}} = {{R rSub {размер 8 {1}} R rSub {размер 8 {2}}} больше {R rSub {размер 8 {1}} + R rSub {размер 8 {2}}}} »{{R rSub {размер 8 {1}} R rSub {размер 8 {2}}} больше {R rSub {размер 8 {1}}}}} {} # = R rSub {размер 8 {2}} слева (R rSub {размер 8 {1}} «>>» R rSub {размер 8 {2}} справа) «.» {}}} {}

Необоснованные результаты

Два резистора, один с сопротивлением 145 Ом, размер 12 {1 «45»% OMEGA} {}

, соединены параллельно для создания общего сопротивления 150 Ом, размер 12 {150% OMEGA} {}

.а) Каково значение второго сопротивления? б) Что неразумного в этом результате? (c) Какие предположения необоснованны или непоследовательны?

Необоснованные результаты

Два резистора, один с сопротивлением 900 кОм, размер 12 {9 «00 к»% OMEGA} {}

, соединены последовательно для создания общего сопротивления 0,500 МОм размером 12 {0 «.» «500 M»% OMEGA} {}

. а) Каково значение второго сопротивления? б) Что неразумного в этом результате? (c) Какие предположения необоснованны или непоследовательны?

(a) -400 кОм размер 12 {+ — «400 к»% OMEGA} {}

(b) Сопротивление не может быть отрицательным.

(c) Считается, что последовательное сопротивление меньше, чем у одного из резисторов, но должно быть больше, чем у любого из резисторов.

Глоссарий

серии
последовательность резисторов или других компонентов, включенных в цепь один за другим
резистор
компонент, обеспечивающий сопротивление току, протекающему через электрическую цепь
сопротивление
, вызывая потерю электроэнергии в цепи
Закон Ома
взаимосвязь между током, напряжением и сопротивлением в электрической цепи: V = размер ИК 12 {V = курсив «ИК»} {}
напряжение
электрическая потенциальная энергия на единицу заряда; электрическое давление, создаваемое источником питания, например аккумулятором
падение напряжения
потеря электроэнергии при прохождении тока через резистор, провод или другой компонент
текущий
поток заряда через электрическую цепь мимо заданной точки измерения
Закон Джоуля
взаимосвязь между потенциальной электрической мощностью, напряжением и сопротивлением в электрической цепи, определяемая по формуле: Pe = IV размер 12 {P rSub {size 8 {e}} = ital «IV»} {}
параллельно
подключение резисторов или других компонентов в электрической цепи таким образом, что каждый компонент получает одинаковое напряжение от источника питания; часто изображается на диаграмме в виде лестницы, где каждый компонент находится на ступеньке лестницы


Эта работа находится под лицензией Creative Commons Attribution 4.0 Международная лицензия.

Вы также можете бесплатно скачать по адресу http://cnx.org/contents/[email protected]

Атрибуция:

Разница между последовательными и параллельными цепями

Под последовательностью цепей понимается цепь, имеющая только один путь, по которому течет ток. В последовательной цепи все компоненты соединены таким образом, что при возникновении неисправности в цепи ток не будет течь через цепь.Ток в последовательной цепи одинаков во всей цепи. С другой стороны, параллельные цепи относятся к схеме с более чем одним путем, по которому протекает ток. В параллельной цепи все компоненты имеют различные ответвления для прохождения тока; таким образом, ток в цепи неодинаков. Прочтите данное руководство, чтобы узнать разницу между последовательными и параллельными схемами.

Что такое последовательная цепь?

Цепь называется последовательной, если ток одинаков во всех компонентах цепи.В последовательных цепях ток имеет только один путь.

В последовательной цепи отношения между током и напряжением прямо противоположны отношениям в параллельной цепи. Ток через каждый элемент серии одинаков и равен току источника (Is). Напротив, напряжение на каждом последовательном элементе (V1, V2. V3) изменяется в соответствии с импедансом (в этом примере, сопротивлением) каждого элемента. Применяется закон Кирхгофа о напряжении (KVL), и напряжение, подаваемое источником (Vs), равно сумме индивидуальных падений напряжения на каждом последовательном элементе.

Преимущество последовательных цепей

  1. Обладает простой и понятной конструкцией.
  2. Быстро не перегревается.
  3. Он имеет более высокое выходное напряжение, поэтому мы можем добавить больше электроприборов.
  4. Он проводит одинаковый ток по всей цепи.

Недостаток последовательной цепи

  1. Если общее количество компонентов в цепи увеличивается, сопротивление цепи увеличивается.
  2. Если неисправность происходит в одной точке, вся цепь разрывается.

Теперь рассмотрим пример последовательной цепи, чтобы понять концепцию.

Найдите ток, протекающий через резисторы R1, R2 и R3.

Учитывая

R1 = 6 Ом

R2 = 6 Ом

R3 = 6 Ом

А V = 36V

Ответ:

Применяя закон Ом в данной цепи, получаем

В = ИК

В = I (R1 + R2 + R3)

I = В / (R1 + R2 + R3)

I = 36 / (6 + 6 + 6)

I = 36/18

I = 2 А

Что такое параллельная цепь?

Параллельная цепь — это цепь с двумя или более двумя путями прохождения тока.В параллельной цепи все компоненты имеют одинаковое напряжение.

В параллельной цепи напряжение на каждом элементе одинаково и равно напряжению источника (Vs), а ток через каждый элемент (I1, I2, I3) изменяется в соответствии с импедансом (в данном примере сопротивление) каждого элемента. Применяется закон Кирхгофа (KCL) _, и общий ток, протекающий от источника (Is), равен сумме отдельных токов, протекающих через каждый параллельный элемент.

Преимущество параллельных цепей

  1. В параллельной цепи, если какой-либо компонент повреждается, ток не прекращается и продолжает течь через другие компоненты; следовательно, другие компоненты работают эффективно.
  2. В параллельной цепи напряжение на всех компонентах одинаково; поэтому все компоненты работают эффективно.
  3. В параллельной цепи вы можете легко подключить или отключить новый компонент, не влияя на работу другого компонента.

Недостаток параллельной цепи

  1. В параллельной схеме нельзя подавать дополнительный источник напряжения.
  2. Параллельная цепь требует большого количества проводов для подключения.

Теперь рассмотрим пример параллельной схемы, чтобы понять концепцию.

Найдите полное сопротивление между точками P и Q

Ответ:

Здесь сопротивление 2 Ом, подключенное параллельно с резистором 3 Ом, дает 6/5 Ом.Теперь резистор 6/5 Ом подключен последовательно с сопротивлением 5 и 4 Ом, поэтому общее сопротивление между точками P и Q = 6/5 + 5 + 4 = 10,2 Ом.

Разница между последовательной и параллельной цепями


Последовательная цепь Параллельная цепь
Цепь называется последовательной, если ток одинаков во всех компонентах цепи. Параллельная цепь — это цепь с двумя или более двумя путями прохождения тока.
Если отказ происходит в одной точке, вся цепь разрывается. В параллельной цепи, если какой-либо компонент повреждается, ток не прекращается и продолжает течь через другие компоненты; следовательно, другие компоненты работают эффективно.
В последовательной схеме все компоненты расположены в одну линию. В параллельной схеме все компоненты расположены параллельно друг другу.
Если более одного резистора подключены последовательно, напряжение на каждом резисторе не будет одинаковым, хотя ток будет одинаковым по всей цепи. Если резисторы подключены параллельно, напряжение на каждом из резисторов будет одинаковым.
Если V — это полное напряжение на всех компонентах в последовательной цепи, оно равно V1 + V2 + V3. Если V — полное напряжение на всех компонентах в параллельной цепи, оно равно V1 = V2 = V3
В последовательной цепи R = R1 + R2 + R3 В параллельной цепи, R = 1 / R1 + 1 / R2 + 1 / R3

Введение в последовательные, параллельные и последовательно-параллельные соединения

Последовательные, параллельные и последовательно-параллельные схемы, их сравнение и применения

Почему параллельное соединение предпочтительнее последовательного соединения?

Сегодня невозможно переоценить использование, применение и важность последовательного и параллельного соединения цепей.Применение последовательного и параллельного подключения цепей можно увидеть в наших домах, школьных залах и в наших уличных фонарях. Одним нажатием кнопки включаются все качели в наших гостиных. некоторые говорят, что у бобов в их домах должны быть разные переключатели.

Что ж, это не волшебство, когда с помощью одного переключателя управляется более трех электрических бобов или нагрузок. Нагрузка — это что угодно, то есть это могут быть приборы, электрические качалки или даже потолочные вентиляторы, которые потребляют электроэнергию при подключении к источнику питания.Электрические бобы, телевизоры, холодильники и т. Д. Можно назвать грузом. Бобы преобразуют электрическую энергию в световую и тепловую форму энергии. Вентиляторы преобразуют электрическую энергию в механическую.

Тип подключения наших потолочных вентиляторов и электрических бобов определит, будут ли они иметь общий выключатель или нет. Последовательное соединение цепи дает нам возможность подключить более двух нагрузок к общему выключателю. Уличные фонари — очень хороший тому пример. Параллельное соединение цепи позволяет нам подключать нагрузки к их индивидуальному переключателю.Подходит как последовательное, так и параллельное соединение, но одно предпочтительнее другого по той или иной причине. Прежде чем говорить о том, почему параллельное соединение предпочтительнее последовательного, давайте вспомним, какие последовательные и параллельные соединения являются первыми.

Последовательная цепь

Последовательная цепь — это цепь, в которой резисторы или нагрузки подключены встык, так что в цепи будет только один путь, по которому протекает электрический ток.Таким образом, когда несколько резисторов соединены последовательно, эффективное сопротивление (общее сопротивление в цепи) получается путем алгебраического сложения отдельных сопротивлений. То есть, если у нас есть резисторы с сопротивлением R1, R2, R3… Rn , соединенные последовательно , то;

R eff = R T = R 1 + R 2 + R 3 +… R n .

При последовательном соединении один и тот же ток течет по всем ветвям цепи, но разное напряжение на нем, что заставляет резисторы иметь разное напряжение на них.На каждом резисторе или нагрузке будет падение напряжения. Приложенное напряжение равно сумме падений напряжения на разных частях цепи. Падение напряжения пропорционально тому, что ток сопротивления одинаков во всей цепи. Когда нагрузки подключаются последовательно, они, как правило, имеют общий выключатель. Такая связь используется в школьных залах, уличных фонарях.

Как подключить фары последовательно?
Использование и применение последовательного соединения

Некоторые люди подключают сигнальные огни в своих домах последовательно, что позволяет использовать общий выключатель.Проблема с этим типом подключения заключается в том, что при возникновении проблемы с нагрузкой другая подключенная система выйдет из строя. Это тип подключения по схеме «все или ничего». Пока нагрузка не получит энергию до того, как она передаст ее другой, и одна из них не выйдет из строя, будет отключение электроэнергии.

Последовательные соединения схем распространены и широко используются в электрическом оборудовании. Нити трубки в небольших радиоприемниках обычно идут последовательно. Устройства управления током всегда подключаются последовательно с устройством, которое они защищают.Предохранители соединены последовательно с устройством, которое они защищают. Автоматическое отопительное оборудование имеет термостат, электромагнитные катушки и предохранительные выключатели, соединенные последовательно с источником напряжения и т. Д.

Недостатки последовательной цепи
  • Разрыв в проводе отказ или удаление любой отдельной лампы приведет к разрыву цепи и приведению к прекращению работы всех остальных, поскольку в цепи протекает только один единственный путь тока.
  • Если добавить в цепь последовательного освещения больше ламп, их яркость снизится.потому что напряжение распределяется по последовательной цепи. Если мы добавим больше нагрузок в последовательную цепь, падение напряжения возрастет, что не является хорошим признаком для защиты электроприборов.
  • Серия
  • Проводка типа «ВСЕ или НЕТ» означает, что все устройства будут работать одновременно или все они отключатся, если произойдет сбой в любом из подключенных устройств в последовательной цепи.
  • Высокое напряжение питания необходимо, если нам нужно добавить дополнительную нагрузку (лампочки, электрические обогреватели, кондиционер и т. Д.) В последовательную цепь.Например, если пять ламп 220 В должны быть подключены последовательно, то напряжение питания должно быть: 5 x 220 В = 1,1 кВ.
  • Общее сопротивление последовательной цепи увеличивается (а ток уменьшается), когда в цепь добавляется дополнительная нагрузка.
  • В соответствии с будущими потребностями, в текущую последовательную цепь следует добавлять только те электроприборы, если они имеют тот же номинальный ток, что и ток в каждой точке последовательной цепи. Однако мы знаем, что электрические приборы и устройства i.е. лампочки, вентилятор, обогреватель, кондиционер и т. д. имеют разные номинальные токи, поэтому их нельзя подключать последовательно для бесперебойной и эффективной работы. Фары, подключенные последовательно
Преимущества последовательного подключения
  • Меньше размер провода кабеля требуется при последовательном подключении.
  • Мы используем для защиты цепи для последовательного подключения предохранителей и автоматических выключателей с другими приборами.
  • Последовательная цепь
  • нелегко получить накладные расходы из-за высокого сопротивления, когда в цепь добавляется дополнительная нагрузка.
  • Срок службы батареи в последовательной цепи больше, чем в параллельной.
  • Это наиболее простой метод электрического подключения, который позволяет легко обнаружить и устранить неисправность по сравнению с параллельным или последовательно-параллельным подключением.

Параллельная схема

Резисторы, нагрузки считаются подключенными параллельно, когда конец каждого из резисторов или нагрузок имеет общую точку или соединение, а другие концы также подключены к общей точке или переходу.Такие схемы известны как параллельные схемы.

Лампочки, подключенные параллельно

В отличие от последовательного подключения, при нахождении общего (эффективного) сопротивления в параллельной цепи берется величина, обратная отдельному сопротивлению. Таким образом, когда несколько сопротивлений соединены параллельно, величина, обратная величине эффективного сопротивления, определяется арифметической или алгебраической суммой величины, обратной величине отдельного сопротивления.

1 / R eff или 1 / R T = 1 / R 1 + 1 / R 2 + 1 / R 3 … 1 / R n .

Параллельное соединение цепи имеет одинаковое напряжение, протекающее по всем ветвям цепи. У разных резисторов свои токи.

Использование и применение параллельного соединения

Параллельное соединение схемы очень распространено. Различные лампы и электроприборы в наших домах подключаются параллельно, так что каждая из ламп или бобов и приборов может работать независимо. Чтобы мы могли управлять отдельными лампами или нагрузками, они должны быть подключены параллельно.

Преимущества параллельной схемы
  • Каждое подключенное электрическое устройство и устройство не зависят от других. Таким образом, включение / выключение устройства не повлияет на другие устройства и их работу.
  • В случае обрыва кабеля или удаления какой-либо лампы все цепи и подключенные нагрузки не разорвутся, другими словами, другие светильники / лампы и электроприборы по-прежнему будут работать без сбоев.
  • Если добавить больше ламп в параллельные цепи освещения, их яркость не будет уменьшаться (как это происходит только в цепях последовательного освещения).Потому что напряжение одинаково в каждой точке параллельной цепи. Короче говоря, они получают то же напряжение, что и напряжение источника.
  • Можно добавить дополнительные осветительные приборы и точки нагрузки в параллельных цепях в соответствии с будущими потребностями, если цепь не будет перегружена.
  • Добавление дополнительных устройств и компонентов не приведет к увеличению сопротивления, но уменьшит общее сопротивление цепи, особенно когда используются устройства с высоким номинальным током, такие как кондиционер и электрические нагреватели.
  • параллельная проводка более надежна, безопасна и проста в использовании. Неисправности в параллельных цепях освещения
Недостатки параллельных подключений
  • Кабель и провод большего размера используются в цепи параллельной проводки освещения.
  • При добавлении дополнительной лампочки в параллельную цепь требуется больше тока.
  • Батарея разряжается быстрее при установке постоянного тока.
  • Схема параллельного подключения более сложна по сравнению с последовательным подключением.

Связанное сообщение: Какая лампа светится ярче при последовательном и параллельном подключении и почему?

Последовательно-параллельные соединения и схемы

Схема не является последовательной или параллельной на следующем рисунке, т.е. это последовательно-параллельная схема. Первые три лампы (B 1 , B 2 и B 3 ) подключены параллельно, а переключатели (S 1 , S 2 и S 3 ) подключены последовательно соответственно.B 7 , B 8 , B 9 и B 10 соединены последовательно друг с другом, в то время как они параллельны первым трем лампочкам (B 1 , B 2 и B 3 ) в то время как переключатели (S5 и S6) параллельно подключены к лампе (B 10 ). Кроме того, лампы (B 4 , B 5 и B 6 ) и выключатель (S 7 ) включены последовательно друг с другом, в то время как они параллельны (B 1 , B 2 и B 3 ) и так далее.

Поскольку схема является комбинацией последовательной и параллельной, мы не можем упростить ток, напряжение, сопротивление и мощность с помощью простого закона Ома. Мы должны применить различные теоремы, такие как теоремы Нортона, Тевенина, теоремы о максимальной передаче мощности и т.д., или упростим схему в основных последовательных и параллельных схемах, чтобы найти все эти величины.

Наиболее распространенная в настоящее время установка бытовой электропроводки с использованием этого метода электропроводки.

Последовательная параллельная световая цепь и соединение

Сравнение последовательного и параллельного подключения

Ниже в данной таблице показаны основные различия между последовательным и параллельным подключением.

9 0598 Чтобы найти электрическую мощность (P)
S Нет Последовательная цепь Параллельная цепь
Ток (I) Ток одинаковый в каждой точке последовательной цепи:

I 1 = I 2 = I 3 =…. I n

Ток в последовательной цепи складывается:

I 1 + I 2 + I 3 +…. I n

Напряжение (В) Напряжение складывается в последовательной цепи:

В 1 + В 2 + В 3 +….V n

Напряжения одинаковы в каждой точке параллельной цепи:

V 1 = V 2 = V 3 =…. V n

Сопротивление (R) и найти (R) Сопротивления складываются в последовательной цепи:

R 1 + R 2 + R 3 +… R n = R eff = R T

Сопротивление делится при увеличении нагрузки в цепи.

1 / R T = 1 / R 1 + 1 / R 2 + 1 / R 3 … 1 / R n

или

I = G 1 + G 2 + G 3 +… G n

Найти ток (I) I = V 1 / R 1 = V 2 / R 2 = V 3 / R 3 = V n / R n I = V 1 / R 1 + V 2 / R 2 + V 3 / R 3 + V n / R n
Найти напряжение (В) V = I 1 R 1 + I 2 R 2 + I 3 R 3 +… I n R n V = I 1 R 1 = I 2 R 2 = I 3 R 3 =… I n R н

P = I 2 R 1 + I 2 R 2 +… I 2 R n

или

P = V 1 2 / R 1 + V 2 2 / R 2 +… V n 2/ R n

P = V 2 / R 1 + V 2 / R 2 +… V 2/ R n

или

P = I 1 2 R 1 + I 2 2 R 2 +… I n 2 R n

Правило делителя тока и напряжения В 1 = В T (R 1 / R T ), V 2 = V T (R 2 / R T ) I 1 = I T (G 1 / G T 900 23), I 2 = I T (G 2 / G T )
Пути прохождения электрического тока Только один путь Два или более пути
Яркость лампы Диммер при добавлении дополнительных ламп (P = V x I) Ярче из-за того же напряжения
Если в цепи случаются обрывы Вся цепь бесполезна Остальная часть цепь будет по-прежнему работать
Состояние батареи Аккумулятор разряжается медленно (емкость аккумулятора в ампер-часах) Аккумулятор разряжается быстро (время работы аккумулятора в ампер-часах и токи)
Приложения Используется для защиты цепи во время подключение предохранителей и автоматических выключателей последовательно с подключенными приборами Используется в большинстве бытовых электропроводок

Преимущества параллельного соединения по сравнению с последовательным соединением

Последовательное соединение — это соединение всех или отсутствующих схем.Это означает, что если одно из устройств выйдет из строя, все другие устройства также выйдут из строя, поэтому этот тип подключения хорош только тогда, когда мы хотим защитить устройство. Когда плавкий предохранитель сгорает, например, из-за высокого тока, устройство его защищает не будет повреждена, потому что ток больше не будет достигать ее. В то время как последовательное соединение является полным или нулевым, параллельное соединение дает вам возможность дать нагрузкам и приборам их индивидуальный переключатель. Параллельное соединение обеспечивает сопротивление протеканию тока по сравнению с последовательным соединением.

Недостатки последовательной схемы освещения

Резисторы сопротивлением 100 Ом и 150 Ом, подключенные параллельно, будут иметь меньшее влияние на электрический ток по сравнению с резисторами 50 Ом и 40 Ом, подключенными последовательно. В электронных устройствах параллельное соединение имеет первостепенное значение. Все элементы в блоке питания подключены параллельно. Параллельное соединение продлевает срок службы электрической энергии. Сами элементы имеют свое внутреннее сопротивление, поэтому, если они были подключены последовательно, некоторая часть энергии будет потеряна, преодолевая внутреннее сопротивление, поскольку при последовательном соединении его влияние выше, чем при параллельном.

Похожие сообщения:

Изучение основ последовательного и параллельного сопротивления [Краткое руководство]

Привет, надеюсь, у тебя хорошая жизнь. В этом посте я поделюсь с вами своими знаниями о последовательном и параллельном сопротивлении. Я поделюсь самыми основными моментами, которым я бы хотел, чтобы кто-нибудь научил меня, когда я изучал основы электроники.

При разработке электронной схемы резистор, вероятно, является наиболее важным и часто встречающимся компонентом. Почти все портативные схемы содержат очень сложные комбинации резисторов.

Следовательно, очень полезно иметь набор правил для определения эквивалентных сопротивлений для некоторых общих и общих схем резистивных элементов. И оказалось, что такие простые правила представляют собой комбинации параллельных серий.

Прежде всего, позвольте мне прояснить основную путаницу. Последовательно-параллельные комбинации аналогичны последовательным и параллельным цепям, но состоят только из резисторных элементов. Фактически, большинство пунктов, которые вы здесь изучите, также применимы к любой последовательной и параллельной схеме.

Таким образом, последовательные и параллельные цепи являются общими терминами, а последовательно-параллельное сопротивление — это конкретные цепи, в которых используются только резисторы.

Сопротивление серии

Резисторы, независимо от их размеров, подключены таким образом, чтобы через каждый из них протекал одинаковый ток. Эквивалентное сопротивление таких подключенных резисторов известно как последовательное сопротивление.

В последовательно соединенных сопротивлениях / резисторах есть только один путь для прохождения электрического тока. Все резисторы соединены в ряд как звенья цепи.При обрыве одного из звеньев цепи вся цепь становится бесполезной. То же самое и с последовательными резисторами, когда один резистор перегорает, вся цепь перестает работать.

Рисунок: Блок-схема последовательного сопротивления (Программное обеспечение: NI Multisim Educational Edition)

Ток остается неизменным в любом узле в цепи последовательного сопротивления. Практически это можно доказать, применив мультиметр и схему на макетной плате. Или просто смоделировав схему (рис. 1) в любом программном обеспечении для моделирования, как я сделал это в NI Multisim.

Как найти полное последовательное сопротивление?

Все резисторы, подключенные последовательно, добавлены для получения эквивалентного последовательного сопротивления. Общая формула приведена ниже:

Чтобы найти эквивалентное последовательное сопротивление, удалите из цепи любой источник напряжения, а затем просуммируйте значения каждого резистора, присутствующего в цепи. Возьмем, к примеру, приведенную выше схему (рисунок: 1).

Сопротивление серии

может быть подключено к источнику тока.Те же шаги выполняются и для текущего источника. Удалите источник тока и просуммируйте значения резисторов. Пример приведен ниже:

Математический расчет:

Схема может иметь любую форму. Но резисторы нужно подключать как звенья цепи. Следующая схема является последовательно соединенной.

Вы можете это доказать?

Можете ли вы найти эквивалентное последовательное сопротивление? (Ответ: 378КОм)

Следует помнить, что последовательно соединенные резисторы имеют только один общий узел друг с другом.Это очень важный момент, который нужно понять. Позже в этом посте я поделюсь примером схемы с комбинацией последовательной и параллельной комбинаций.

Тогда это будет немного запутать. Так что обратите на это пристальное внимание, потому что этот момент поможет нам идентифицировать последовательную комбинацию резисторов в сложной цепи.

Параллельное сопротивление

Резисторы, независимо от их размеров, подключены таким образом, чтобы на каждом резисторе было одинаковое напряжение.Эквивалентное сопротивление таких подключенных резисторов известно как параллельное сопротивление.

Или параллельное сопротивление можно определить как резисторы , подключенные параллельно, если узлы на обоих концах резисторов одинаковы. В параллельно соединенных резисторах (параллельное сопротивление) есть разные пути для прохождения электрического тока. Величина протекающего тока зависит от номиналов резисторов.

Параллельно подключенные резисторы

При параллельном подключении эквивалентное сопротивление всегда меньше, чем у наименьшего отдельного резистора.Например, эквивалентное сопротивление вышеупомянутой цепи всегда будет меньше 1 кОм. И напряжение на каждом резисторе составляет 12 В, поскольку все резисторы подключены к тем же узлам, что и резисторы 12 В.

Я говорил о резисторах , включенных параллельно, если узлы на обоих концах резисторов одинаковые. позвольте мне показать общие узлы в приведенной выше схеме.

Как найти полное параллельное сопротивление?

В параллельно соединенных резисторах эквивалентное сопротивление равно сумме обратных величин каждого резистора.Общая формула эквивалентного параллельного сопротивления приведена ниже:

Найдем значение полного сопротивления для указанной выше цепи.

Удалите источник напряжения и затем введите значения резисторов в приведенную выше общую формулу.

Ответ — меньше самого маленького отдельного резистора.

Приведем еще один пример:

Подскажите, как эта схема вообще параллельна?

Конечно, это параллельно включенная схема.

Это соответствует критерию, резисторы включены параллельно, если узлы на обоих концах резисторов одинаковы.

Форма сбила нас с толку, правда? Но все же это резистивная цепь, включенная параллельно. Дело в том, что я говорю. Неважно, какую форму может иметь схема, вы должны знать базовую концепцию того, как идентифицировать параллельные резисторы в сложной схеме.

Мы можем перерисовать приведенную выше схему в чистом виде, как показано ниже:

Быстрый трюк, чтобы найти эквивалентное параллельное сопротивление

Когда все параллельно подключенные резисторы имеют одинаковые значения, эквивалентное параллельное сопротивление можно найти, просто разделив одно и то же значение на общее количество резисторов.Общая формула может быть записана так:

Давайте применим этот быстрый трюк на вышеуказанной схеме.

Было бы хорошо, если бы вы потратили некоторое время, чтобы доказать это, используя общую формулу для параллельного сопротивления. Будет интересно увидеть, что оба метода дадут одинаковый ответ.

Различия между последовательным и параллельным сопротивлением

Я не знаю, сколько разницы между указанными сопротивлениями в целом. Ниже приведены различия, которые я узнал со временем.

1. При последовательном сопротивлении один и тот же ток протекает через каждый резистор, в то время как при параллельном сопротивлении каждый резистор имеет одинаковое напряжение на нем.

2. В последовательном сопротивлении полное напряжение равно падению напряжения на каждом резисторе. См. Следующую схему:

Общее приложенное напряжение составляет 12 В (аккумуляторный источник), что означает, что общая сумма напряжений на каждом резисторе должна быть равна 12 В.

Попробуем решить эту схему вручную.

Резисторы включены последовательно. Общее последовательное эквивалентное сопротивление можно найти, используя общую формулу для последовательного сопротивления.

Используя закон Ома, полный ток можно определить следующим образом:

Мы рассчитали полный ток. Чтобы найти падение напряжения на каждом резисторе, умножьте общий ток на номинал резистора (закон Ома).

Мы получили те же значения, что и при моделировании Multisim. Суммирование всех рассчитанных напряжений закончится на 12 В.Вы можете доказать это, добавив их самостоятельно с помощью калькулятора.

3. При параллельном сопротивлении полный ток равен сумме токов, протекающих через каждый резистор.

Общий ток (22,0 мА) равен сумме токов, протекающих от каждого резистора. Вы можете доказать это, сложив их вместе с помощью калькулятора. Не путайте с отрицательным знаком. Игнорируй это.

4. В последовательном сопротивлении, если один резистор не работает должным образом, остальная часть цепи также не работает.Параллельно выход из строя одного резистора не влияет на всю схему.

5. Падение напряжения в последовательном сопротивлении пропорционально размеру резистора. В то время как при параллельном подключении количество тока, протекающего в каждом резисторе, зависит от размера резистора. Увеличение размера резистора увеличивает падение напряжения, а также ток.

Зачем вообще нужны эти комбинации?

Есть очень важные применения последовательных и параллельных комбинаций.Прямо сейчас я делюсь тем, что знаю, и буду обновлять по мере того, как узнаю об этом больше.

Я работал над схемой, и мне понадобился резистор 2 кОм, которого у меня не было в то время. Я много искал в своей маленькой лаборатории, но все тщетно. Затем я собрал два резистора 1 кОм, чтобы получился резистор 2 кОм. Я сделал это, поставив последовательно два резистора 1кОм.

Думаю, вы поняли. Последовательное и параллельное соединение могут быть очень полезны, если у вас нет с собой резистора точного номинала. Это также может быть полезно, если вы разрабатываете схему и вам нужен резистор, которого нет на ближайшем рынке.Вы можете получить желаемое значение резистора, играя с последовательными и параллельными комбинациями резисторов.

Эти комбинации также находят применение в конструкции вольтметра и амперметра соответственно.

Сопротивление серии

используется в качестве ограничителя тока в цепях. Его можно использовать как делитель напряжения для светодиодной схемы. Возможно, вы видели, что у каждого светодиода есть резистор, включенный последовательно. Задача резистора — ограничить ток, протекающий через светодиод.

Пример

Давайте рассмотрим пример, чтобы еще больше прояснить нашу концепцию последовательного параллельного соединения.Схема в примере имеет смешанные параллельные и последовательные комбинации. Поэтому, пожалуйста, обращайте пристальное внимание и старайтесь решать на каждом этапе.

Я рекомендую захватить с собой блокнот, калькулятор и хорошую ручку. И давай попробуем решить со мной.

Сводка

  • Резисторы подключены таким образом, что через каждый резистор протекает одинаковый ток, тогда эквивалентное сопротивление резисторов называется последовательным сопротивлением.
  • В серии ток остается неизменным независимо от размеров резисторов.А это возможно только тогда, когда резисторы соединены как звенья цепи.
  • В параллельно соединенных резисторах напряжение на каждом резисторе остается неизменным независимо от номинала каждого резистора.

Надеюсь, этот пост был вам полезен. Вот что я знаю о последовательном и параллельном сопротивлении. И я буду продолжать обновлять этот пост по мере того, как узнаю что-то новое по упомянутой теме.

Большое спасибо за чтение и хорошей жизни.


Прочие полезные сообщения

резисторов в последовательном и параллельном соединении сетей

резисторов в последовательном и параллельном соединении

резисторы могут быть соединены отдельно друг от друга последовательно или только параллельно.Некоторые схемы резисторов состоят из комбинации последовательных и параллельных цепей для создания более сложных схем. Эти схемы обычно известны как схемы со смешанными резисторами. Несмотря на то, что в этих схемах объединены последовательные и параллельные цепи, метод расчета эквивалентного сопротивления не изменился. Основные правила отдельных сетей, такие как «одинаковый ток течет через последовательно подключенные резисторы» и «одинаковое напряжение на резисторах, подключенных параллельно», применимы к смешанным схемам.

Пример схемы смешанных резисторов показан ниже

Он состоит из четырех резисторов R1, R2, R3 и R4 в комбинации схем смешанных резисторов. Напряжение питания равно V, а общий ток, протекающий в цепи, равен I. Ток, протекающий через резисторы R2 и R3, равен I1, а ток, протекающий через резистор R4, равен I2.

Здесь последовательно соединены резисторы R2 и R3. Следовательно, применяя правило последовательно соединенных резисторов, эквивалентное сопротивление R2 и R3 дается как

R A = R2 + R3

Здесь RA — эквивалентное сопротивление R2 и R3

Теперь резисторы R2 и R3 могут заменить на одиночный резистор РА.Полученная схема показана ниже.

Теперь резисторы RA и R4 соединены параллельно. Следовательно, применяя правило параллельных резисторов, эквивалентное сопротивление RA и R4 составляет

R B = R A × R4 / (R A + R4)

Здесь RB — эквивалентное сопротивление RA. и R4

Теперь мы можем заменить резисторы RA и R4 одним резистором RB. После замены резисторов полученная схема показана ниже.

Теперь схема состоит всего из двух резисторов. Здесь также последовательно соединены резисторы R1 и RB. Следовательно, применяя правило последовательных резисторов, общее эквивалентное сопротивление цепи определяется как

R EQ = R1 + R B

Здесь R EQ — полное эквивалентное сопротивление цепи. Теперь резисторы R1 и R B можно заменить на один резистор R EQ .

Окончательная эквивалентная схема для вышеуказанной сложной схемы показана ниже.

Несмотря на то, что они выглядят сложными, схемы смешанных резисторов могут быть сведены к простой схеме, состоящей только из одного источника напряжения и одного резистора, следуя простым правилам последовательного включения резисторов и параллельных резисторов.

Последовательные и параллельные резисторы Пример

Рассчитаем эквивалентное сопротивление для схемы ниже, состоящей из 7 резисторов R1 = 4 Ом, R2 = 4 Ом, R3 = 8 Ом, R4 = 10 Ом, R5 = 4 Ом, R6 = 2 Ом и R7 = 2 Ом. Напряжение питания 5 В.

Теперь резисторы R6 и R7 соединены последовательно. Если эквивалентное сопротивление последовательностей R6 и R7in равно Ra, тогда

Ra = R6 + R7 = 2 + 2 = 4 Ом

Результирующая цепь сокращается до показанной ниже.

В приведенной выше схеме резисторы Ra и R5 соединены параллельно. Следовательно, эквивалентное сопротивление Ra и R5 составляет

R b = (R a × R 5 ) / (R a + R 5 ) = (4 × 4) / (4 + 4 ) = 2 Ом.

Тогда упрощенная схема показана ниже.

В этой схеме резисторы R4 и R b соединены последовательно.

Rc = R4 + R b = 10 + 2 = 12 Ом.

Теперь мы можем заменить резисторы R4 и R b на резистор Rc, как показано ниже.

В приведенной выше схеме резисторы R2 и R3 снова соединены последовательно. Если Rd эквивалентно сопротивлению R2 и R3, тогда

Rd = R2 + R3 = 4 + 8 = 12 Ом.

Эквивалентная схема:

Здесь резисторы Rc и Rd соединены параллельно. Пусть Rp будет эквивалентным сопротивлением Rc и Rd, включенных параллельно. Тогда

R p = (R c × R d ) / (R c + R d ) = (12 × 12) / (12 + 12) = 6 Ом.

В результате получается схема

Здесь резисторы R1 и Rp соединены последовательно. Пусть R EQ будет эквивалентным сопротивлением этой комбинации.

Тогда

R EQ = R1 + Rp = 4 + 6 = 10 Ом.

Это эквивалентное сопротивление цепи. Следовательно, данная схема может быть окончательно перерисована как

Ток в цепи можно рассчитать по закону Ома

I = V / R EQ = 5/10 = 0,5 A

Резисторная сеть

Вычислим эквивалентное сопротивление для сложной резисторной схемы.

Схема ниже состоит из десяти резисторов от R1 до R10, соединенных последовательно и параллельно.

Значения сопротивлений, указанных в цепи, указаны в Ом (Ом), а напряжение питания — в Вольтах (В).

Здесь последовательно соединены резисторы R9 и R10. Пусть R A — эквивалентное сопротивление этой комбинации.

Следовательно, R A = R9 + R10 = 3 + 3 = 6 Ом.

Схема после замены R9 и R10 на R A :

В этой схеме резисторы R8 и R A соединены параллельно.Тогда эквивалентное сопротивление R8 и R A равно

R B = (R8 × R A ) / (R8 + R A ) = (6 × 6) / (6 + 6) = 3 Ω.

Теперь заменив R8 и R A на R B , мы получим следующую схему.

В этой схеме резисторы R7 и R B соединены последовательно.

R C = R7 + R B = 9 + 3 = 12 Ом.

Эквивалентная схема после замены R7 и R B на R C :

Понятно, что резисторы R6 и Rc соединены параллельно.Если R D является эквивалентным сопротивлением этой комбинации, тогда

R D = (R6 × Rc) / (R6 + Rc) = (12 × 12) / (12 + 12) = 6 Ом.

Схема с R¬D, ​​заменяющим R6 и Rc:

Теперь резисторы R4 и RD соединены последовательно. Если RE эквивалентное сопротивление R4 и RD, тогда

R E = R4 + R D = 6 + 6 = 12 Ом.

Результирующая сокращенная схема после замены R4 и R D на R E составляет

В этой схеме резисторы R5 и R E соединены параллельно.

Пусть R F будет эквивалентным сопротивлением R5 и R E , включенных параллельно.

Тогда

R F = (R5 × R E ) / (R5 + R E ) = (12 × 12) / (12 + 12) = 6 Ом.

Упрощенная схема показана ниже.

Здесь резисторы R2 и R3 включены последовательно. Если RG является эквивалентом этой комбинации, то

R G = R2 + R3 = 4 + 2 = 6 Ом.

После замены R2 и R3 на RG схема преобразуется в

Резисторы RF и RG включены параллельно.

Пусть R T будет эквивалентом этой комбинации.

Тогда R T = (R F × R G ) / (R F + R G ) = (6 × 6) / (6 + 6) = 3 Ом.

Теперь резисторы R1 и RT включены последовательно. Если REQ — полное эквивалентное сопротивление цепи, то REQ = R1 + RT = 3 + 3 = 6 Ом.

Наконец, вышеуказанная сложная схема может быть перерисована следующим образом

Общий ток в цепи может быть рассчитан по закону Ома

I = V1 / R EQ = 6/6 = 1 A

Следовательно, любой комплекс Резистивную цепь, состоящую из числа резисторов, соединенных в комбинации как последовательной, так и параллельной комбинаций, можно уменьшить, сначала определив простые параллельные ветви резистора и ветви последовательного резистора.Рассчитывается эквивалентное сопротивление этих простых ветвей, и ветви заменяются эквивалентным резистором. Этот процесс снижает сложность схемы. Продолжая этот процесс, мы можем заменить сложную резистивную цепь одним резистором.

Существуют некоторые сложные резистивные схемы, которые нельзя свести к простым схемам, просто применяя правила последовательных резистивных комбинаций и параллельных резистивных комбинаций. Такие схемы, как аттенюаторы T-Pad и некоторые сложные резистивные мостовые схемы, являются примерами таких сложных резистивных схем.Чтобы упростить эти сложные резистивные схемы, следует использовать другой подход.

Некоторые сложные резистивные цепи можно уменьшить, используя закон Кирхгофа по току и закон Кирхгофа по напряжению.

Найти токи и напряжения в сложной резистивной цепи, просто используя закон Ома, может быть невозможно. Для такого типа цепей будут полезны правила Кирхгофа для цепей.

Законы Кирхгофа для цепей основаны на концепции сохранения тока и энергии в цепи.Есть два закона Кирхгофа. Первый — это закон тока Кирхгофа, который касается тока в узле, а второй — закон напряжения Кирхгофа, который касается напряжения в замкнутой цепи.

Разное

Добавить комментарий

Ваш адрес email не будет опубликован. Обязательные поля помечены *